FP515 Retirement Savings and Income Planning

अब Quizwiz के साथ अपने होमवर्क और परीक्षाओं को एस करें!

James and Hannah have determined that they will need a monthly income of $6,000 at the beginning of each month during retirement. They expect to receive Social Security retirement benefits amounting to $3,500 per month. Over the 12 remaining years of their preretirement period, they expect to generate an average annual after-tax investment return of 8%; during their 25-year retirement period, they want to assume a 6% annual after-tax investment return. How much do they need to save at the end of each month to build the necessary retirement fund? A) $1,653 B) $1,621 C) $1,785 D) $1,611

$1,621 The monthly retirement income need is not specified as "today's dollars," and no inflation rate specified; therefore, it is assumed that the $2,500 net monthly income need represents retirement dollars, and the retirement period income stream is level. First, to calculate the lump sum needed at the beginning of retirement, discount the stream of monthly income payments at the investment return rate: Set calculator to the Begin mode and 12 periods per year, then input the following: PMT=$2500/ N=300 (25*12)/ I=6%/ FV=0 --> PV = -$389,957 Set calculator to END FV=$389,957/ N=144 (12*12)/ I=8%/ PV=0 --> PMT= $1,621

Justin's employer grants him 1,000 shares of restricted stock worth $10 per share at the time of the grant. Justin pays nothing for the grant. The terms of the restriction require him to remain employed by his company for an additional five years. When he meets the five-year requirement, the stock is trading for $25 per share. What is the amount Justin must report as compensation (W-2) income for the year in which he meets the five-year requirement? A) $12,500 B) $25,000 C) $0 D) $10,000

$25,000 Justin must report the value of the stock as compensation income in the year in which the stock is no longer subject to a substantial risk of forfeiture. Therefore, he must report $25,000 ($25 × 1,000 shares) of income for the current year.

Tammy doesn't begin her Social Security benefit at FRA (age 67), instead opting to delay while collecting delayed retirement credits. When she reaches age 69, however, she encounters financial difficulty and must file for her benefits. How much will her payment have increased? A) 24% B) 16% C) 5.5% D) 8%

16% Tammy will receive two years of delayed retirement credits. Each year she will accrue an 8% payment increase, for a total of a 16% increase.

Two years ago, Marilyn, an executive vice president with ABC Bank, was granted a nonqualified stock option (NQSO) to purchase 100 shares of ABC stock at $10 per share. When the fair market value (FMV) of the stock reached $30 per share this year, she exercised the option. She is in the 24% marginal income tax bracket. If Marilyn chooses to use a cashless exercise, how many shares of ABC stock will she own after the transaction? A) 30 B) 49 C) 51 D) 100

51 The answer is 51. The cost to exercise the options is $1,000 (100 shares × $10 per share). Also, because the option is an NQSO, Marilyn will have to pay ordinary income taxes of $480 on the bargain element [($30 per share FMV - $10 per share exercise price) × 100 shares × 24% tax rate]. Marilyn's total cost to exercise the option is $1,480. Because this is a cashless exercise, enough shares of stock must be sold to cover Marilyn's cost of the option. Therefore, 49 shares of the stock must be sold ($1,480 ÷ $30 per share FMV). Marilyn will have 51 shares remaining (100 − 49).

If provisional income exceeds the thresholds given, then a maximum of ___________ of Social Security benefits are subject to taxation. A) 50% B) 85% C) 20% D) 37%

85% If provisional income exceeds the stated threshold, a maximum of 85% of the excess amount is taxable as ordinary income.

Ross, age 75, works for Financial Strategies, Inc. The company has a long-established retirement plan. The plan does not require an actuary or Pension Benefit Guaranty Corporation (PBGC) insurance, but the employer is required to make annual mandatory contributions to each employee's account. What type of retirement plan was established by Financial Strategies? A) Nonqualified deferred compensation plan B) Cash balance pension plan C) Traditional defined benefit pension plan D) Money purchase pension plan

A money purchase pension plan requires annual mandatory employer contributions to each employee's account, does not require an actuary, and does not require PBGC insurance. The other choices are incorrect: - A cash balance pension plan requires an actuary and PBGC insurance (LO 2.2.1). - Nonqualified deferred compensation plans are not required to be established for each employee's account. They are for selected, highly paid executives. - A traditional defined benefit pension plan requires the services of an actuary annually as well as PBGC insurance (LO 2.1.1).

David has been calculating his retirement savings needs. He has $250,000 of assets set aside now. His goal is to have $800,000 in 15 years. How much more does he need to save, assuming a 7% annual after-tax rate of return throughout this period? (Round to the nearest dollar.) A) $110,242 B) $689,758 C) $289,957 D) $550,000

A) $110,242 Calculating the future value of David's current retirement assets: −$250,000 (PV) 15 years from now (N) at a 7% return (I/YR) equals $689,758 (FV). This leaves a difference of $110,242 ($800,000 − $689,758) to be saved by David.

Paul estimates he will need $75,000 of annual income in today's dollars when he retires 10 years from now. He assumes a 3% annual rate of inflation, a 5% after-tax rate of return on his investments, and a 20-year retirement period. Using the serial payment approach, how much will Paul need to save in a single annual payment at the end of the first year to fund his retirement need? A) $118,578.00 B) $100,794.00 C) $120,880.49 D) $115,124.27

A) $118,578.00

Which of these retirement plans would be appropriate for a general partnership with stable cash flows? A) Age-weighted profit-sharing plan B) Employee stock ownership plan (ESOP) C) Section 403(b) plan (TSA) D) Stock bonus plan

A) Age-weighted profit-sharing plan

Which of these statements falsely describes a concept related to nonqualified deferred compensation? A) An example of substantial risk of forfeiture provisions would be the employee's loss of rights to the plan benefits because of disability or premature death. B) The availability of deferred compensation plan funds to the employee, without substantial restriction, generally results in constructive receipt. C) The employee's receipt of anything that can be assigned a cash value results in economic benefit and taxation. D) Substantial risk of forfeiture exists when the employee's receipt of deferred compensation benefits is contingent upon performance of substantial services in the future.

A) An example of substantial risk of forfeiture provisions would be the employee's loss of rights to the plan benefits because of disability or premature death. Disability or premature death does not create a substantial risk of forfeiture.

Which one of the following is a correct statement about the amount of Social Security retirement benefits available when a fully insured worker's retirement benefit begins at her full retirement age (FRA)? A) At his FRA, the worker's spouse will receive 50% of the worker's primary insurance amount (PIA). B) A 63-year-old spouse of the retired worker will receive 50% of her PIA. C) The worker will receive 80% of her PIA. D) If the spouse has attained his FRA and is entitled to benefits on his earning record, he will receive the lower of 100% of his own PIA or 50% of the PIA based on her earnings.

A) At his FRA, the worker's spouse will receive 50% of the worker's primary insurance amount (PIA). The spouse, at his full retirement age (age 65 to age 67), will receive 50% of the worker's PIA unless the spouse's Social Security benefit is higher based on his own earnings.

Which of these objectives might be met by an employer's use of a nonqualified retirement plan? A) Employers want to boost morale for all workers. B) The employer needs to bring executive retirement benefits up to desired levels by adding a second tier of benefits on top of the qualified plan. C) The employer desires a plan in which benefits may legally discriminate in favor of highly paid employees. D) The employer desires an alternative to the qualified plan because of the complexity of legislative changes.

A) Employers want to boost morale for all workers. Deferred comp plans are design to boost the morale and commitment of selected employees, not all workers.

Which of these are characteristics of phantom stock plans? I. Company stock is issued to the employee as a reward for meeting performance goals. II. The employee becomes vested in theoretical shares according to a written agreement. III. The plan is a way to reward employees without giving them actual ownership in the company. IV. Upon retirement or some other event, the employee receives the vested value of his account in cash. A) II, III, and IV B) III and IV C) I and II D) I, II, and III

A) II, III, and IV Under a phantom stock plan, no stock is actually issued to the participant. The plan is a means for companies to give employees a bonus tied to performance and reward employees without giving them actual ownership in the company. The employee becomes vested in theoretical shares according to a written agreement. Upon retirement or termination, the employee receives the vested value of his account in cash either as a lump sum or in installments.

The 20% mandatory withholding requirement applies to distributions from all of the following except A) IRAs. B) qualified plans. C) Section 403(b) plans. D) Section 457 plans.

A) IRAs.

Which of these retirement plans can be integrated with Social Security? A) Money purchase pension plans B) Salary reduction SEPs (SARSEPs) C) Employee stock ownership plans (ESOPs) D) Savings incentive match plan for employees (SIMPLEs)

A) Money purchase pension plans The answer is money purchase plan. Profit-sharing plans, Simplified Employee Pension (SEP) plans, money purchase plans, and defined benefit pension plans may be integrated with Social Security. Employee stock ownership plans (ESOPs), savings incentive match plan for employees (SIMPLEs), and salary reduction SEPs (SARSEPs) are not permitted to use integration. Also, employee elective deferrals and employer matching contributions cannot be integrated.

Conway Enterprises, Inc., wishes to set up an equity-based compensation plan that offers the following benefits to employees: Tax advantages Immediate sale of the stock Voting rights when the stock is awarded Shares granted to an employee at no cost or at a bargain price Which of the following would be the best choice for the company? A) Restricted stock plan B) Junior class shares C) Severance pay plan D) Stock appreciation rights

A) Restricted stock plan

Social Security began as a program to provide retirement income but has been expanded to provide all of the following income except: A) survivor benefits to spouse caring for a child under 19. B) survivor benefits to spouses at age 60‒61. C) survivor benefits to children under age 19 if still in high school or lower. D) disability.

A) survivor benefits to spouse caring for a child under 19. Survivor benefits are provided to a spouse caring for a child under 16 or disabled. Age 16 is not "sweet 16" for the surviving spouse because when the youngest child turns 16, the survivor benefits ceased for the widow(er) caring for the deceased's child.

Which of the following retirement plan alternatives would allow Tomas, a 51-year-old self-employed business owner with $50,000 of self-employment income, the greatest deductible contribution while providing him with only a small cash flow commitment each year based on 2023 plan contribution limits? A) Safe harbor 401(k) plan B) Defined benefit pension plan C) SEP plan D) Profit-sharing plan

A) Safe harbor 401(k) plan The defined benefit pension plan requires annual funding. The SEP and the profit-sharing plans both limit contributions to 25% of earned income, which converts to 20% because Tomas is self-employed. Although a SEP or profit-sharing plan provides tremendous flexibility, a Section 401(k) plan could provide the same flexibility and provide a significantly higher benefit. Tomas is self-employed and, therefore, must first reduce his Schedule C net income by the deductible portion of his self-employment tax and then multiply the result by 20% to determine the maximum profit-sharing contribution. Using 2023 plan contribution limits as a guide, Tomas's business can contribute approximately $9,294 as an employer contribution to either a SEP plan or a profit-sharing plan. With a safe harbor 401(k) plan, he can increase the contribution by deferring $30,000 ($22,500 salary deferral plus $7,500 as a catch-up contribution in 2023). Making the 401(k) a safe harbor plan would decrease some of the administrative costs. Additionally, he would have to fund the required employer match, but this would be small compared to the other listed plans.

While Section 403(b) (tax-sheltered annuity plan or TSA) plans are an excellent source of retirement savings, they do have some disadvantages, such as A) Section 403(b) plans must comply with the actual contribution percentage (ACP) test for employer matching contributions. B) actual deferral percentage (ADP) testing causes Section 403(b)/TSA plans to be relatively costly and complex to administer. C) investments are limited to mutual funds, individual stocks, and annuities. D) account balances at retirement age are guaranteed to be sufficient to provide adequate retirement amounts for employees who entered the plan at later ages.

A) Section 403(b) plans must comply with the actual contribution percentage (ACP) test for employer matching contributions. Section 403(b) plans must comply with the actual contribution percentage (ACP) test for employer matching contributions. Section 403(b)/TSA plan investments are limited to mutual funds and annuities. Although the ADP test does not apply, Section 403(b)/TSA plans must comply with the ACP test for matching contributions. One way to remember that Section 403(b) plans must pass ACP testing and not ADP testing is that Section 403(b) plans are for 501(c)(3) organizations. The (c) in 501(c)(3) is like the "C" in ACP testing. Nondiscrimination testing causes Section 403(b)/TSA plans to be relatively costly and complex to administer. Account balances at retirement age may not be sufficient to provide adequate retirement amounts for employees who entered the plan at later ages. Section 403(b)/TSA plan investments are limited to mutual funds and annuities.

Which of the following retirement plans, maintained by an eligible employer, would also permit the employer to establish a SIMPLE IRA? A) Section 457 plan B) Money purchase pension plan C) Section 403(b) plan D) Traditional Section 401(k) plan

A) Section 457 plan To establish a SIMPLE (IRA or Section 401(k)), an employer cannot generally maintain another retirement plan. However, a Section 457 plan is technically a nonqualified plan and, therefore, does not constitute a plan for purposes of establishing a SIMPLE of any type.

Which of these is false regarding qualified preretirement survivor annuities (QPSAs)? A) The QPSA payable to the surviving spouse must be equal to the benefit that would have been payable to the participant. B) If the participant dies before the earliest retirement date, QPSA payments are to begin no later than the month in which the participant would have reached the earliest retirement age under the plan. C) To waive the QPSA benefit for a married participant's spouse, the participant and the spouse must consent on a notarized written form. D) If the participant dies before retiring but after the earliest retirement date, QPSA payments must be the actuarial equivalent of not less than half of the participant's vested account balance as of the date of the participant's death.

A) The QPSA payable to the surviving spouse must be equal to the benefit that would have been payable to the participant. QPSA payments are not required to equal the benefit that would have been payable to the participant at retirement, but must be the actuarial equivalent of not less than half of the participant's vested account balance as of the date of the participant's death.

All of the following are primary assumptions in any retirement needs calculation EXCEPT: A) the number of dependents a client will have at retirement. B) the client's age at retirement and anticipated life expectancy. C) the anticipated annual rate of inflation. D) the projected rate of annual investment return.

A) the number of dependents a client will have at retirement. The answer is the number of dependents a client will have at retirement. There are three primary assumptions that are made in any retirement needs analysis calculation. They are the anticipated annual rate of inflation; the projected rate of annual investment return; and the client's age at retirement and anticipated life expectancy.

A local competitor company has been aggressively recruiting Chris. If Chris leaves Gilmore Glass, which statement regarding the tax status of any outstanding loan from his Section 401(k) plan is CORRECT if Chris cannot arrange to pay off the loan? A) the outstanding loan may be subject to an additional 10% excise penalty. B) The loan balance continues to be tax free because Chris is essentially borrowing from himself. C) Chris may roll over the outstanding loan to an IRA and continue paying the loan under the same rules as before. D) The outstanding loan is not subject to income tax if not repaid.

A) the outstanding loan may be subject to an additional 10% excise penalty. No one can roll a retirement plan loan to an IRA and keep the payments and term the same. By using a plan loan offset, Chris could lower the amount of the defaulted loan by making payments into his new employer's plan or an IRA up to the due date (including extensions) of his tax return for the year of the retirement plan loan default. The loan becomes fully taxable if not repaid and may be subject to an additional 10% excise penalty. Since he would be separating from service, the defaulted loan amount would be subject to the QPLO rules.

Required minimum distributions from a qualified plan to a plan participant must be calculated using the Uniform Lifetime Table in all cases except: A) when the designated beneficiary is the participant's spouse and the spouse is more than 10 years younger than the participant. B) when the designated beneficiary is a child under the age of 16. C) when there is more than one designated beneficiary. D) when there is no designated beneficiary.

A) when the designated beneficiary is the participant's spouse and the spouse is more than 10 years younger than the participant. The Uniform Lifetime Table must be used to calculate required minimum distributions under a qualified plan or IRA unless the designated beneficiary is the participant's spouse and the spouse is more than 10 years younger than the participant, in which case the actual joint life expectancy is used.

ERISA requirements for qualified plans include: A) reporting and disclosure. B) coverage and vesting. C) participation and fiduciary requirements. D) all of these.

All of these

An employer is permitted to make what level of contributions to an employee stock ownership plan (ESOP)? A) An ESOP is a defined contribution plan that limits employer contributions to 25% of covered payroll. B) An ESOP is a defined benefit plan. Thus the contribution limit is the amount required to fund the projected benefits. C) An ESOP is a stock bonus plan that permits a maximum contribution to a leveraged ESOP of 27% (rather than the 25% that would otherwise apply). D) An ESOP is a stock bonus plan that permits a maximum contribution to a non-leveraged ESOP of 35% (rather than the 25% that would otherwise apply).

An ESOP is a defined contribution plan that limits employer contributions to 25% of covered payroll. Defined contribution plans limit employer contributions to 25% of covered payroll. With a leveraged ESOP, the employer may deduct both the principal (up to 25% of compensation paid to covered employee-participants) and additional amounts paid as interest as the loan is paid off. An ESOP is not a defined benefit plan.

Paul estimates he will need a $75,000 annual income in today's dollars when he retires 10 years from now. He assumes a 3% annual rate of inflation, a 5% after-tax rate of return on his investments, and a 20-year retirement period. What lump-sum amount should Paul accumulate over the next 10 years to support his retirement income need? A) $2,015,880 B) $1,689,612 C) $1,257,227 D) $1,657,429

B) $1,689,612

Carol has an additional retirement need of $30,000 annually in today's dollars. She will retire in 15 years and projects a retirement period of 20 years. Carol believes she can achieve a 6% after-tax rate of return and is assuming a 4% annual rate of inflation. She has accumulated $175,000 toward her retirement plan. What lump-sum amount should Carol have accumulated over the next 15 years to support her retirement income need? A) $503,707 B) $907,144 C) $873,553 D) $732,144

B) $907,144 Carol's total retirement fund needed to support her desired standard of living is $907,144, calculated as follows: Carol's first year retirement income need is $54,028. PV = -$30,000 I/YR = 4 N = 15 FV = $54,028 The total capital required to support this need for 20 years is $907,144. In BEGIN mode (the client will make annual withdrawals at the beginning of each year) PMT = $54,028 N = 20 I/YR = 1.9231 [(1.06 ÷ 1.04) - 1] × 100 PVAD = -$907,144

A reduced Social Security retirement benefit is available for all retirees as early as which age? A) 65 B) 62 C) 60 D) 59½

B) 62 Individuals may begin receiving a reduced benefit at age 62. Full benefits begin between ages 65 and 67 depending on the year of birth. Individuals who delay receiving benefits beyond their full retirement age receive increased benefits.

Tamu Enterprises, Inc., would like to set up an equity-based compensation plan that has the following features: - Shares that are a separate class of common stock - Shares that are usually convertible into common shares - Shares that allow employees to purchase company stock below fair market value - Shares with rights that are subordinate to regular common stock shares Which of the following would be the best choice for the company? A) Restricted stock B) Junior class shares C) Incentive stock options D) Stock appreciation rights

B) Junior class shares Junior class shares are a separate class of common stock. The voting, liquidation, and dividend rights of junior class shares are subordinate to regular common stock shares. Junior class shares are generally convertible into common shares of stock upon specified requirements, such as achieving certain years of service or performance goals. Conversion can be automatic or at the employee's discretion.

Which of these persons could make tax-deductible contributions to a traditional IRA regardless of their modified adjusted gross income (MAGI)? A) A person who participates in a Section 401(k) plan B) A person who participates in a Section 457 plan C) A person who participates in a Section 403(b) plan D) A person who participates in a SEP IRA

B) A person who participates in a Section 457 plan The answer is a person who participates in a Section 457 plan. A person who participates in a qualified plan, SEP IRA, or Section 403(b) plan may not be able to make tax-deductible IRA contributions if the participant's MAGI exceeds certain limits. Participation in a Section 457 plan does not subject a person to these limitations.

Which one of the following is a CORRECT statement about the amount of Social Security retirement benefits available when a fully insured worker's retirement benefit begins at full retirement age (FRA)? A) If the full retirement age (FRA) spouse also is entitled to benefits on his or her earning record, the ultimate benefit is the lesser of 100% of the spouse's own PIA or 50% of the worker's PIA. B) At their own FRA, the worker's spouse will receive 50% of the worker's primary insurance amount (PIA). C) A 63-year-old spouse of the retired worker will receive 50% of the worker's PIA. D) The worker will receive 80% of his or her primary insurance amount (PIA).

B) At their own FRA, the worker's spouse will receive 50% of the worker's primary insurance amount (PIA). The spouse, at his or her FRA, will receive 50% of the worker's PIA unless the spouse's Social Security benefit is higher based on his or her own earnings. At full retirement age the worker will receive 100% of his or her PIA and the worker who retires at prior to their FRA would receive a reduced benefit based on how many months early the Social Security retirement benefits were started. The reduction for workers is 5/9 of 1% for each of the first 36 months and 5/12 of 1% for each month in excess of 36 months prior to FRA. The spousal benefit would be less than 50% for a spouse who is not at FRA when spousal retirement benefits started. The 50% of PIA is reduced by 25/36 of 1% for each of the first 36 months (plus 5/12 of 1% for each month in excess of 36 months prior to FRA) the spouse is under FRA when benefits begin. A spouse who is at FRA and entitled to benefits on their own record would receive the higher of 100% of their own PIA or 50% of their spouse's PIA.

Amy, age 63 and single, expects to earn $28,000 from her job at the flower shop. She is also receiving Social Security benefits. What impact will these earnings have on her Social Security benefits? A) Her benefit will be reduced because her provisional income is in excess of the threshold and she is under full retirement age. B) Her benefit will be reduced because her earned income is in excess of the threshold and she is under full retirement age. C) There will be no reduction because her provisional income is under the threshold. D) There will be no reduction because her earned income is under the threshold.

B) Her benefit will be reduced because her earned income is in excess of the threshold and she is under full retirement age. Her earned income is above the threshold for people receiving Social Security retirement benefits who are ages 62 through the year before FRA. Provisional income is a term used for taxation of Social Security benefits, not for reduction of Social Security benefits.

Whose Social Security benefit is included in the calculation of the maximum family benefit for those eligible for benefits based on a retired worker's fully insured status? I. The retired worker's benefit II. The retired worker's former spouse II. A dependent child's benefit IV. A caregiver spouse's benefit for caring for a qualified disabled child A) I, II, III, and IV B) I, III, and IV C) III and IV D) I and II

B) I, III, and IV Only Statement II is incorrect. Benefits paid to a former spouse of a covered worker are not considered in the application of the family maximum benefit limit.

Joe, age 52, has just started a consulting company. He currently employs six people, who range in age from 22 to 31 years old. Joe estimates the average employment period for his employees will be approximately three years and would like to implement a retirement plan that will favor older participants while including an appropriate vesting schedule. In addition, Joe would like the employees to bear the risk of investment performance within the plan. Which of the following plans is most appropriate for Joe's company? A) SEP plan B) Target benefit pension plan C) Cash balance pension plan D) SIMPLE 401(k)

B) Target benefit pension plan A target benefit pension plan is likely most appropriate. It would permit Joe to favor older participants and allow for a vesting schedule. A cash balance pension plan does not favor older participants and provides employees with a guaranteed rate of return on investments (thus, not transferring the risk of investment performance to the employees). SEP plans and SIMPLE 401(k) plans both provide for 100% immediate vesting of employer contributions.

Terri is 58 years old. Her company anticipates that the next several years will be profitable, but not extravagant. Terri's actuary informed her that a defined benefit plan would cost her $100,000 per year. Terri is only willing to put a total of $50,000 into the company's retirement plan. Most of her workers are 20-35 years old. What type of qualified retirement plan should Terri select? A) Defined benefit plan B) Target benefit plan C) Money purchase plan D) Profit-sharing plan

B) Target benefit plan The best choice for Terri is a target benefit plan. The key is that the owner is older than the workers but cannot afford a defined benefit plan. Also, the firm can afford the mandatory annual contributions.

Bernie and Tim, both age 53, are partners in a computer software consulting firm. They have 20 employees whose average age is 25 and average length of employment is three years. The firm is highly profitable and enjoys stable cash flows. Of the following retirement plan options, which is best suited to the partners' business? A) Section 403(b) plan B) Traditional defined benefit pension plan C) A stock bonus plan D) An eligible Section 457 plan

B) Traditional defined benefit pension plan Of the listed plans, only a traditional defined benefit pension plan is available to Bernie and Tim's business. A Section 403(b) plan may only be adopted by a Section 501(c)(3) organization. An eligible Section 457 plan may only be adopted by a private, tax-exempt organization or a state or local governmental organization. A partnership cannot use a stock bonus plan because this form of business does not issue stock. Notice how subtlety the form of business was introduced in the question. The form of business is very important in selecting a retirement plan.

All of the following statements regarding the capital preservation approach to retirement needs analysis calculations are correct EXCEPT: A) this approach assumes the client will spend the income from his retirement fund and leave the principal intact. B) this approach is the most commonly used approach in making retirement needs analysis calculations. C) this approach is most appropriate for affluent clients. D) a person using this approach will need to accumulate a larger retirement fund than if he had used the capital utilization approach.

B) this approach is the most commonly used approach in making retirement needs analysis calculations. The capital preservation approach assumes the client will spend the income from his retirement fund but leave the principal intact. This approach requires the client to accumulate a larger retirement fund than the capital utilization approach does, and it is generally appropriate for more affluent clients. Most retirement needs analysis calculations use the capital utilization approach.

Which of the following workers must have Social Security and Medicare taxes withheld from their earnings? I. A household worker, unrelated to the employer and age 25, who is paid $10,000 II. An agricultural worker who paid in excess of a specified threshold A) II only B) Both I and II C) Neither I nor II D) I only

Both I and II

Ray died this year at age 73, and his wife, Mary, 55, is the designated beneficiary on his Roth IRA. Ray's Roth IRA was established three years ago. Which of the following statements is CORRECT? I. Ray was not subject to required minimum distributions from his Roth IRA during his lifetime. II. If Mary chooses to distribute the entire balance of the Roth IRA this year, a portion of the distribution may be subject to regular income tax. A) I only B) Both I and II C) II only D) Neither I nor II

Both I and II Statement I is correct. Roth IRA owners are not required to take minimum distributions from their IRA during their lifetime. Statement II is correct. Mary would need to hold the IRA two additional years to satisfy the five-year holding period required for tax-free distributions. If distributed this year, any amount attributed to earnings would be subject to regular income tax. Any amounts attributed to contributions would not be income taxed. Nor would the contribution amount be subject to the 10% early withdrawal penalty because contribution amounts withdrawn are never subject to the 10% EWP. Any conversion amount withdrawn would not be subject to income taxes because the income taxes were paid when the conversion was made. Thus, conversion amounts that are withdrawn from Roth IRAs are never subject to income taxes again because that would be double taxation. However, this particular Roth IRA is three years old, so withdrawals of conversion amounts would be subject to the 10% EWP rules. In this case, however, there would be no 10% EWP because the distribution is due to a death.

Which of these statements regarding the structure of a nonqualified deferred compensation plan is CORRECT? I. A pure deferred compensation arrangement uses some portion of the executive's current compensation to fund the promised compensation benefit. II. Using the salary continuation approach, the plan is funded with money that the employer has set aside from current earnings to benefit the executive.

Both I and II The answer is both I and II. A pure deferred compensation arrangement uses some portion of the executive's current compensation to fund the promised compensation benefit. Using the salary continuation approach, the plan is funded with money that the employer has set aside from current earnings to benefit the executive.

Which of the following describe differences between a tax-advantaged retirement plan and a qualified plan? I. IRA-funded employer-sponsored tax-advantaged plans may not incorporate loan provisions. II. Employer stock distributions from a tax-advantaged plan do not benefit from NUA tax treatment. A) Neither I nor II B) I only C) II only D) Both I and II

Both I and II Both I and II are correct. IRA-funded employer-sponsored tax-advantaged plans are SEPs, SARSEPs, and SIMPLE IRAs.

Thad and Debra, both age 48, are married and will file a joint return. Their 2023 modified adjusted gross income is $120,000 (including Thad's $95,000 salary). Debra had no earned income of her own. Neither spouse was covered by an employer-sponsored retirement plan. What is the total maximum deductible contribution Thad and Debra may make to a traditional IRA this year? A) $5,200 B) $0 C) $13,000 D) $6,500

C) $13,000 The answer is $13,000. Because neither Thad nor Debra participates in an employer-sponsored retirement plan, they can contribute and deduct $6,500 each for 2023. While Debra has no earned income, a spousal IRA may be established and funded based on Thad's compensation.

our client is considering how to pay for the last year of her child's college tuition. Her 401(k) balance is $87,000 and she has never borrowed from her retirement account before. How much of a loan could she take from her 401(k)? A) $87,000 B) $20,250 C) $43,500 D) $50,000

C) $43,500 Retirement account loans are generally 50% of the vested balance minus the highest loan amount in the previous 12 months. In this case, there is no previous loan amount in the last 12 months, so she has access to $43,500 ($87,000 ÷ 2).

Maryellen, age 63, is receiving Social Security retirement benefits. She also works part time, and her earnings are $10,000 more than the earnings limit. Her Social Security retirement benefits this year will be reduced by A) $0. B) $7,500. C) $5,000. D) $10,000.

C) $5,000. Maryellen has not reached full retirement age, so she is subject to the retirement earnings test. Her Social Security benefits will be reduced by $1 for every $2 in earnings over the applicable limit.

The Jones Corporation has a profit-sharing plan with a 401(k) provision. The company matches dollar-for-dollar up to 5%. Pedro makes $150,000 and defers 5% into the 401(k) for 2023. The Jones Corporation has had a banner year and is considering a large contribution to the profit-sharing plan. What is the most that could be contributed to Pedro's profit-sharing account this year? A) $22,500 B) $58,500 C) $51,000 D) $66,000

C) $51,000 The maximum allowed employer profit sharing contribution in this case for 2023 is $51,000. The Section 415 annual additions limit for 2023 is $66,000. However, Pedro has already contributed $7,500, and this amount has been matched. Thus, $15,000 has already gone toward the $66,000 annual additions limit for 2023.

Norman and Brenda (both age 36) are married taxpayers filing jointly. Norman earned $132 this year, and Brenda earned $100,000. Brenda is an active participant in the qualified plan offered by her employer, and she contributed $1,500 to her IRA for this tax year. How much, if any, can be contributed to a spousal IRA and deducted for Norman for this year? A) $132 B) $0 C) $6,500 D)$5,190

C) $6,500 The maximum deductible contribution to a spousal IRA for Norman is $6,500, and the deductible amount phases out at AGI of $218,000-$228,000 for Norman, who is the nonactive participant spouse.

A variable universal life insurance benefit provided as a part of a qualified defined contribution plan is considered incidental so long as the employer's contributions toward that benefit are no more than ___ of aggregate contributions to the plan. A) 50% B) 35% C) 25% D) 100%

C) 25% The answer is 25%. If term or universal life insurance is used as a funding vehicle in a qualified plan, it is considered incidental if the aggregate premiums paid for the life insurance do not exceed 25% of the employer's aggregate contributions to the plan.

Reed, age 45, has come to you for help in planning his retirement. He works for a manufacturing company, where he earns a salary of $75,000. Reed would like to retire at age 65. He feels this is a realistic goal because he has consistently earned 9% on his investments and inflation has only averaged 3%. Assuming he is expected to live until age 90 and he has a wage replacement ratio of 80%, how much will Reed need to have accumulated on the day that he retires to adequately provide for his retirement lifestyle? A) $1,863,311 B) $1,408,595 C) $1,160,240 D) $1,490,649

D) $1,490,649 (Make sure calc. is in begin mode for step #2 calc.)

Which of these may be taxable to a profit-sharing plan participant even when there is no distribution from the plan? A) Employer contributions B) Investment earnings C) Life insurance premiums for a policy held by the plan D) Employee contributions

C) Life insurance premiums for a policy held by the plan

Claude's ex-wife, Sara, has secured a qualified domestic relations order against his Section 401(k) plan. What are Sara's rollover options if she takes a lump-sum distribution of her share of Claude's retirement plan account? A) Sara may roll over the distribution to her own qualified plan, Section 457 plan, or an IRA, but not to a Section 403 (b) plan. B) Sara may roll over the distribution to her own qualified plan, Section 403(b) plan, or Section 457 plan, but not to an IRA. C) Sara may roll over the distribution to her own qualified plan, Section 403(b) plan, Section 457 plan, or IRA. D) Sara may roll over the distribution to her own qualified plan, Section 403(b) plan, or an IRA, but not to a Section 457 plan.

C) Sara may roll over the distribution to her own qualified plan, Section 403(b) plan, Section 457 plan, or IRA. Sara may roll over the distribution to her own qualified plan, Section 403(b) plan, Section 457 plan, or IRA.

How are Social Security benefits funded? A) Liability is calculated, and FICA taxes are assessed as if Social Security were a defined benefit plan. B) Special taxes are collected and deposited to a trust fund that is only allowed to disburse the benefits. C) Special taxes are collected from workers and used to pay benefits to those workers who are retired. D) Benefits are paid by income taxes, which are increased to offset shortfalls.

C) Special taxes are collected from workers and used to pay benefits to those workers who are retired. Social Security and FICA taxes are withheld by the employer. The total is 15.3% of payroll; the employer pays 7.65% and the employee pays 7.65%. While payroll taxes go into the Social Security Trust Fund, excess funds are "loaned" to the government for other government spending. Other than IOUs from the government, the trust fund doesn't have assets. Benefits are not paid by income taxes. They are paid from a general account funded by payroll deductions and employer matching.

ABC Company would like to establish a retirement plan incorporating the following objectives: I. Attract and retain employees. II. The employer will make all contributions to the plan with company stock. III. The plan must integrate with Social Security. IV. What type of retirement plan best suits ABC's objectives? A) Salary reduction SEP (SARSEP) B) Employee stock ownership plan (ESOP) C) Stock bonus plan D) Money purchase pension plan

C) Stock bonus plan A stock bonus plan would accomplish ABC Company's objectives. The others are not the best plan for these reasons: Money purchase pension plans only allow for 10% company stock. ESOPs cannot be integrated with Social Security. New SARSEPs can no longer be established.

If the Dawsons were to both die in a car accident on April 1, 2023, how long would their beneficiaries be able to stretch their IRAs? A) Their full IRA accounts would need to be distributed by December 31, 2023. B) Their accounts would be split between their two children and each would have his own RMD. C) Their full IRA accounts would need to be distributed by December 31, 2028. D) Their accounts would be immediately disgorged to their estate.

C) Their full IRA accounts would need to be distributed by December 31, 2028. An estate cannot be a designated beneficiary. Thus, each of their IRAs would have to take the entire balance by the end of the fifth year following the year of death. 2023 is the year of death. 2024 is the first year following the year of death. That makes 2028 the fifth year following the year of death.

he primary reason inflation rates for retirees/senior citizens may exceed historical averages is that A) federal and state income taxes are often higher during retirement. B) vacation travel costs may increase during retirement. C) health care costs may increase during retirement. D) many people adopt luxurious lifestyles during retirement.

C) health care costs may increase during retirement. The rising cost of health care is the primary reason senior citizens may experience an inflation rate that exceeds the historical average of 3% per year. This can be especially true if long-term care is needed.

Mary is 66 years old and receives full old-age benefits from Social Security—in her case, $1,200 per month. Her husband, Ralph, age 67, who has not worked enough quarters outside the home to be covered in his own right, receives 50% of what Mary receives each month ($600). Assume that Mary dies tomorrow. What will Ralph's Social Security benefit be? A) $600 + $1,200, or $1,800 B) $600 C) $0 D) $1,200

D) $1,200 The $600 spousal benefit stops, and Ralph will begin receiving 100% of Mary's old-age Social Security benefit. This survivor benefit is not reduced because Ralph has reached his full retirement age (FRA) when his wife died.

Lucy's employer grants her 500 shares of restricted stock worth $5 per share at the time of the grant. The terms of the restriction require Lucy to remain employed by her company for three more years. After the three-year restriction has been met, the stock is trading at $32.50 per share. How much is Lucy required to report as compensation (W-2) income for the year? A) $0 B) $2,500 C) $13,750 D) $16,250

D) $16,250 The answer is $16,250. When the stock is no longer subject to a substantial risk of forfeiture, the value of the stock is taxed as compensation (W-2) income to the employee; therefore, she must report $16,250 ($32.50 × 500) of income for the year.

Use the following information about Kevin and Cindy, a married couple, to answer the question that follows. Kevin and Cindy are both 38 and are planning to retire at 65. They estimate that they will need a lump sum of $2.4 million at retirement to provide the income stream required during their retirement years. They project that their current assets will grow to a value of $1.9 million at the first year of retirement. They feel they can earn a 6% after-tax return on their investments and would like to assume that inflation will average 4% over the long term. They would like to increase their annual savings amount each year as their incomes increase. They would like to assume a 30-year period of retirement. They have no heirs and would like to assume the worst-case scenario: that they will use up all their assets during retirement. They would like you, their financial planner, to determine the annual serial saving re

D) $5,157 First, the capital utilization method was used to calculate their shortfall of $500,000 ($2,400,000 - $1,900,000). This shortfall must be deflated at a 4% rate over the 27 years until retirement (age 38 to age 65). The deflated value of additional savings needed at retirement is $173,408. Next, determine the amount that they need to save at the end of the first (current) year. This amount will then be increased at the inflation rate each year during the preretirement period. The variables are: FV $173,408 (the deflated value of additional savings needed at retirement), 27 years until retirement, 1.9231% inflation-adjusted yield using 6% after-tax return, and a 4% inflation rate. The result of $4,960 must then be adjusted for inflation since the savings deposit is assumed to be made at the end of the year. Thus, $5,157 is the first year's serial savings need. The other choices are incorrect for the following reasons: $4,960 is the result when one assumes the payments are made at the beginning of the year, i.e., $4,960 × 1.0 $7,849 is the level payment amount using $500,000, 27 years, and 6% $8,626 is the result when it is assumed that $173,408 is a present value

If the client's business objectives are to reduce income tax, reward executive employees, retain and recruit employees, and reduce employee turnover, which plan selection approach could address these issues? A) Profit-sharing only approach B) Pension plan only approach C) None of these D) A pension plan or a profit-sharing plan

D) A pension plan or a profit-sharing plan

Charles (age 38) has just died. He has been credited with the last 30 consecutive credits of Social Security coverage in the last 30 quarters since he left school and began full-time employment. He had never worked before leaving school. Which of these persons are eligible to receive Social Security survivor benefits as a result of Charles's death? A) Charles's child, Dawn, age 19 B) Charles's widow, Maggie, age 38 C) Charles's dependent mother, Betty, age 60 D) Charles's child, Bill, age 16

D) Charles's child, Bill, age 16 Dawn is too old under the rules, Maggie does not have a child under age 16 for whom she is caring, and Betty is ineligible because she is not age 62 or older. Only Bill is eligible to receive a dependent or surviving child's benefit.

Under the Social Security system, immediate survivor income benefits based on a deceased worker's primary insurance amount and coverage are available to which of the following persons? I. A surviving spouse, age 55, caring for the worker's 13-year-old child II. Unmarried, dependent children under age 18 III. Unmarried children who become disabled before age 22 IV. A surviving divorced spouse, age 62, who has not remarried and was married to the decedent for more than 10 years A) I only B) II and III C) III and IV D) I, II, III, and IV

D) I, II, III, and IV Each of these persons is eligible for survivor's benefits.

What is the earliest age at which a currently insured worker may claim Social Security retirement benefits based on her own work history? A) Age 67 B) Any age over age 59½, but benefits will be reduced C) Age 62 D) Retirement benefits are not available for a currently insured worker

D) Retirement benefits are not available for a currently insured worker The worker must be fully insured to qualify for retirement benefits based on her own work history. Reduced spousal retirement benefits are available as early as age 62. Reduced survivor benefits are available as early as age 60 unless the surviving spouse is disabled, then reduced survivor benefits are available as early as age 50.

Several years ago, Greener Grass Company implemented a traditional defined benefit plan. According to the plan document, the employer must contribute an annual amount that will provide the employees with a specified benefit at retirement. Which of these events would be expected to decrease the employer's annual contribution to a traditional defined benefit pension plan using a percentage for each year of service benefit formula? A) Inflation is higher than expected. B) Benefits are cost of living adjusted. C) Forfeitures are less than anticipated. D) investment returns of the plan are greater than expected. Forfeitures are less than anticipated would result in increased contributions.

D) investment returns of the plan are greater than expected. Forfeitures are less than anticipated would result in increased contributions. The answer is investment returns of the plan are greater than expected. Forfeitures are less than anticipated would result in increased contributions. Inflation would likely cause salaries and plan expenses to increase, thereby causing contributions to increase. Likewise, benefits that are adjusted for the cost of living would result in greater employer contributions, not less.

Sam was born in 1954, so his full retirement age (FRA) is 66. If he begins receiving Social Security benefits early, the amount of his monthly Social Security checks will be A) reduced until he reaches age 72. B) reduced until he reaches age 66. C) reduced by the amount of the benefits that is above a taxable base amount. D) reduced for the rest of his life.

D) reduced for the rest of his life. Early retirees will have their retirement benefits reduced throughout the remainder of their lives.

A government employer would choose to establish a Section 457 plan for all of the following reasons except A) income tax credit for certain taxpayers regarding elective contributions. B) no early withdrawal penalty on distributions. C) tax-deferred growth. D) tax deductibility of contributions.

D) tax deductibility of contributions. Because Section 457 plans are sponsored by tax-exempt entities, deductibility of plan contributions is not an issue.

Which of the following statements describing how qualified pension plans differ from SEP and SIMPLE plans is(are) CORRECT? I. Qualified plan rules provide greater flexibility in the number and makeup of the employees covered by the plan than do the rules pertaining to SEP and SIMPLE plans. II. Participants must be fully and immediately vested in the contributions to qualified plans, but SEP and SIMPLE plans are permitted to have vesting schedules. A) Both I and II B) II only C) Neither I nor II D) I only

I Only The answer is I only. Statement II is incorrect because participants must be fully and immediately vested in the contributions to SEPs and SIMPLEs. Qualified plans can include vesting schedules.

Which of these statements regarding employee stock purchase plans (ESPPs) is CORRECT? I. The option price must be at least 85% of the FMV of the stock at the time of the grant. II. If an employee separates from service, she must wait three years to exercise the options in an ESPP. III. No employee can acquire the right to buy more than $100,000 of stock per year, valued at the time the option is granted.

I Only The answer is I only. The shares acquired must be held by the employee at least two years from the date of the grant of the option and one year from the date of the exercise. Employees owning more than 5% of the corporation are not allowed to participate in an ESPP. No employee can acquire the right to buy more than $25,000 of stock per year, valued at the time the option is granted.

The employer bears the investment risk for which of the following retirement plans? I. Cash balance pension plan II. SIMPLE 401(k) plan III. Age-weighted profit-sharing plan IV. Money purchase pension plan

I Only The employer bears the investment risk for the cash balance pension plan. For the remaining plans, the employee bears the investment risk.

Which of the following statements regarding the anticipated effective income tax rate a planner should use for required retirement plan distributions is CORRECT? I. The projected rate should be based only on a blend of current federal and state marginal income tax rates. II. The projected rate should be based only on current federal marginal income tax rates. III. Precisely predicting future income taxes is not feasible. IV. A planner should only use before-tax rate of return assumptions on retirement plan distributions. A) IV only B) II only C) II and III D) I and III

I and III Before retirement, the planner may use a before-tax rate of return in the assumptions, particularly if tax-advantaged savings vehicles (such as a traditional or Roth IRA) are used in the planning process. However, at the time of either optional or required retirement plan distributions, the client's anticipated effective income tax rate is very important. This rate should be a blend of the client's federal and state marginal income tax rates but should be projected based only on current rates because precisely predicting future income tax rates is not feasible.

Which of the following correctly describe the earnings limitation on Social Security benefits? I. Retirement or survivor benefits may be reduced $1 for every $2 earned over the specified limit by recipients who are age 62 through the year just prior to the year they attain full retirement age (FRA). II. Retirement or survivor benefits may be reduced $1 for every $3 earned over the specified limit by recipients who are at FRA. III. Recipients who have attained their FRA may earn any amount without any reduction in their Social Security benefits. IV. One-half of unearned (investment) income may be considered in the reduction of Social Security benefits. A) I and III B) I, II, and III C) II and IV D) I only

I and III Earnings limitations apply to Social Security benefit recipients who are age 62 through the year just prior to the year containing their full retirement age (FRA); no reductions are imposed on recipients who have attained their FRA. Investment income is not considered in determining the earnings reduction.

Which of the following transactions by a qualified plan's trust are subject to the unrelated business taxable income (UBTI) rules? I. A trust obtains a low-interest loan from an insurance policy it owns and reinvests the proceeds in a certificate of deposit paying a higher rate of interest. II. A trust constructs a residential apartment building and receives rent from the tenants. III. A trust owns vending machines located on the employer/plan sponsor's premises. IV. A trust owns raw land that it rents to an oil and gas developer.

I and III Options I and III are subject to the UBTI rules because income from almost any type of property will be taxable as UBTI if the property has been acquired with borrowed funds (e.g., a life insurance policy loan), and income from the operation of a business (e.g., vending machines) is subject to UBTI treatment. Options II and IV are not subject to the UBTI rules because real property rents are statutorily exempt from UBTI. Hence, the rent from the building and raw land is not subject to UBTI treatment.

Which of these statements, with respect to making a Section 83(b) election on restricted stock, is CORRECT? I. An employee who receives restricted stock may elect under Section 83(b) to recognize the income immediately rather than wait until the substantial risk of forfeiture expires. II. The Section 83(b) election must be made within one year of receiving the restricted stock. III. If the election is made, the employee immediately includes as long-term capital gain the fair market value of the stock at time of receipt, less any amount paid for the stock. IV. If the employee makes the election and then forfeits the stock, the employee is not allowed a deduction or refund of tax on previously reported income. A) I only B) I, III, and IV C) II, III, and IV D) I and IV

I and IV The answer is I and IV. The Section 83(b) election must be made within 30 days of receiving the restricted stock. The employee recognizes ordinary income and not long-term capital gain when the election is made. The 83(b) election is sometimes referred to as the "Las Vegas election" because it is irrevocable. You pay income taxes upfront. This income cannot be recovered later if the deal goes sour.

Which of the following should a businessowner accomplish before considering the adoption of a retirement plan? I. Purchase personal and business liability insurance. II. Establish cash reserves sufficient to cover potential emergencies. III. Ensure the business has sufficient cash flow to support ongoing funding of the plan.

I, II and III A businessowner (or a person planning for his own retirement) should accomplish all of these objectives before considering the adoption of a retirement plan.

Joaquin was granted enough nonqualified stock options (NQSOs) to purchase 10,000 shares of Nova, Inc., stock at $10 per share two years ago. He is fully vested in the options. Which of the following are income tax consequences if Joaquin gifts the NQSOs to a qualified charity before the option's exercise date? I. Joaquin does not recognize any gain on the transfer. II. Joaquin will receive a charitable income tax deduction in the year of the gift. III. Joaquin will be taxed at ordinary income rates if the charity exercises the options while he is living.

I, II and III Joaquin does not recognize gain on the transfer date. He will have ordinary income if he is living when the charity exercises the options. Joaquin will be allowed a charitable income tax deduction on the date of transfer because he is fully vested in the options.

The tax treatment of junior class shares of stock include which of these? I. The employee is not taxed when the junior class shares are purchased and issued. II. The employee is not taxed when the junior class shares are converted to common shares. III. The employee is taxed when the junior class shares are converted to common shares and then sold. IV. When the common shares are sold, the employee incurs ordinary income taxes and FICA based on the sales price. A) III and IV B) I, II, and III C) I, II, III, and IV D) I and II

I, II and III The answer is I, II, and III. The employee is neither taxed on the date the junior class shares are issued, nor when the junior class shares are converted to common shares. Upon sale of the common shares, the employee is taxed on the capital gain (the difference between the sales price of the stock and the employee's basis in the stock). The employee's basis is the amount paid for the junior class shares.

Which of these statements regarding stock appreciation rights (SARs) are CORRECT? I. The executive to whom they are granted does not need to have cash available to exercise the stock appreciation rights (SARs). II. They are similar to phantom stock, except that with SARs, the employee can choose when to exercise the right to share in the appreciation of the closely held company's stock. III. Like phantom stock, they require cash to create and dilute ownership. IV. They are used heavily by closely held businesses. A) II, III, and IV B) I and III C) II and IV D) I, II, and IV

I, II and IV Cash does not dilute company ownership. A primary feature of SARs is that dilution of ownership is avoided because the employee gets cash instead of an ownership stake..

For an executive to retain the tax benefits of deferred compensation: I. she must not have constructive receipt of the funds. II. the funds must not be set aside in a specific account for the executive. III. she must have substantial risk of forfeiture. IV. the employer must make an unsecured promise to pay the compensation. A) III and IV B) I and II C) I, II, III, and IV D) I, II, and III

I, II, III and IV

Which of the following are types of Section 401(k) plans? I. Roth 401(k) II. SIMPLE 401(k) III. Traditional 401(k) IV. Safe harbor 401(k)

I, II, III and IV

Increases in which of the following can push a retiree's rate of inflation higher than the overall annual rate? I. Housing costs II. Travel goals III. Food prices IV. Personal buying habits

I, II, III and IV All of the statements are correct. The rate of inflation for retiree-senior citizens may well be higher than a 3% per year average annual rate. This is primarily because of the rising cost of health care in the United States. In addition, a retiree's actual rate of inflation may vary significantly from the CPI figure because of regional housing or food prices, personal buying habits, and travel goals.

Johnson Services Inc. has been in operation for eight years and has been profitable for the past three years. Due to competitive pressures, the company will undergo an expansion of its workforce over the next five years—from the current 17 employees to a projected 36 employees. Tim Johnson, the owner, is interested in installing a qualified retirement plan to attract employees and reduce the company's tax burden. Tim is 38 years of age, and his financial advisor has recommended that he save 10% of his salary each year for a retirement fund. Which of the following statements describe a key factor that affects the selection of a qualified retirement plan—in this case, making a defined contribution plan more appropriate than a defined benefit plan? I. Tim's interest in attracting employees to the company II. The uncertainty of future cash available for plan contributions, given the planned growth of the company III

I, II, III and IV Either a defined contribution plan or a defined benefit plan could be attractive to potential employees. However, Tim's interest in attracting employees to the company, the uncertain cash flow outlook, Tim's young age, and his retirement savings need of only 10%, all point in the direction of defined contribution plans.

Section 401(k) plans must have automatic survivor benefits (QJSAs and QPSAs) unless: I. the plan provides that, upon the participant's death, the vested account balance will be paid in full to the surviving spouse. II. the plan is not a direct or indirect transferee of a plan to which the automatic survivor annuity requirements apply. III. the participant elects to receive payment as a lump-sum distribution. IV. the participant does not elect payments in the form of a life annuity. A) III and IV B) I, II, and III C) I, II, III, and IV D) I and II

I, II, III and IV For a participant to have elected to receive a lump-sum distribution, the spouse must sign a spousal consent form in front of a notary or an authorized plan representative.

A retirement plan participant died before retirement, and there is a cost basis associated with her account. Which of the following state the beneficiary's income tax liability due to death benefits paid from a qualified plan as either life income or installment payments? I. When the benefits are from life insurance, the cash value portion is taxed under the annuity rules. II. If the benefits are not related to life insurance, the employee's cost basis becomes the cost basis for the beneficiary. III. If the benefits are not related to life insurance, the includible amount is generally taxed as ordinary income. IV. When the benefits are from life insurance, the amount deemed to be pure insurance is excludible from gross income.

I, II, III and IV The annuity rules govern the taxation of the cash value portion of the benefits. The beneficiary assumes the employee's tax basis if the death benefits are not from life insurance. The beneficiary's cost basis is used to determine the taxable portion of a distribution that does not come from life insurance. Pure insurance death benefits are received income tax free.

Dillon's 401(k) has just been changed to allow Roth elective deferrals. Which of the following does a Roth 401(k) plan have in common with a Roth IRA? I. They both are phased out for higher income earners. II. Withdrawals after age 59½ are tax-free if the account is at least five years old. III. Contributions are not deductible. IV. They are both subject to RMDs for retirees starting after attaining age 72. A) I and IV B) II and IV C) II and III D) I and II

II and III

Which of the following correctly describe characteristics of unrelated business taxable income (UBTI) in qualified plan trusts? I. A qualified plan trust that incurs UBTI will be disqualified by the IRS. II. UBTI generally will result from a qualified plan trust's directly operating a business that is not related to the purpose of the trust. III. Common stock that is purchased through the use of debt results in UBTI. IV. Real property that is purchased through the use of debt results in UBTI.

II and III The rules regarding UBTI reflect the need for a qualified plan trust to focus on providing retirement benefits, not on operating an unrelated business or incurring debt. Generally, the trust is prohibited from incurring debt; thus, common stock purchased on margin will result in UBTI on the associated dividends and/or gains. (Real estate is the exception; debt-financed real estate generally is not subject to UBTI.) However, the bottom line is that UBTI just means the usually tax-free retirement trust would owe income tax on these activities. UBTI would not disqualify the retirement plan.

Which of the following statements is(are) CORRECT regarding the Social Security Windfall Elimination Provision (WEP)? I. The WEP is designed to reduce Social Security retirement benefits for any worker who is eligible for retirement benefits from multiple employers. II. An example of a worker who may be subject to the WEP is one who works for a government agency or a foreign employer. III. The WEP applies to all Social Security benefits for which a family member or dependent of a covered worker may qualify. IV. In the case of a worker subject to the WEP, the provision is applied by reducing the formula used to calculate a worker's primary insurance amount (PIA), resulting in a lower benefit. A) I only B) II and IV C) II, III, and IV D) I and III

II and IV The WEP does not apply simply if a worker is eligible for retirement benefits from multiple employers. It may apply to a worker who qualifies for a Social Security retirement or disability benefit but who also earns a pension from an employer who does not withhold Social Security taxes. An example of such an employer could be a state or local government agency that had opted out of the Social Security retirement system; a public school employee is some states; or an employer in a foreign country. Under this provision, the formula used to calculate a worker's PIA is reduced, resulting in a lower benefit. Social Security benefits are designed to pay a higher percentage of career average earnings for lower-paid workers than are paid to higher earning workers. Statement III is incorrect because the WEP does not apply to spousal or survivors benefits under Social Security. The Government Pension Offset would apply to Social Security spousal or survivor benefits when the non-Social Security worker is the spouse or the survivor of a fully covered retired or deceased worker.

Sally, age 60, has received a $50,000 distribution from $100,000 she is to receive from her ex-husband's (who is age 55) qualified plan account under a qualified domestic relation order (QDRO). Which of the following statements is CORRECT regarding the QDRO and the distributed funds? I. Irrespective of the plan document, Sally may demand an immediate cash distribution of the remaining funds from the plan trustees. II. She may roll over the $50,000 distribution into an IRA. III. Sally's ex-husband is not subject to an early distribution penalty in the execution of the QDRO. IV. Sally may be required to leave the remaining funds with the plan trustee until the earliest time for distributions under the plan. A) II, III, and IV B) I, II, III, and IV C) II and III D) I, II, and III

II, III and IV A trustee may not be forced to distribute assets from a plan unless the plan document allows for it.

When she retired at age 64, Lauren received a lump-sum distribution from her employer's stock bonus plan. The fair market value of the employer stock contributed to her account was $200,000 at the time of contribution. At the time of the distribution, the employer stock in Lauren's account had a fair market value of $300,000. Six months later, Lauren sold the stock for $310,000. Which of the following statements regarding the sale of Lauren's stock is(are) CORRECT? I. The $300,000 distribution is taxed at the long-term capital gain rate. II. Lauren has a $10,000 short-term capital gain when the stock is sold. III. There was no income tax liability incurred when the stock was contributed to the plan. IV. The net unrealized appreciation (NUA) on the stock is $100,000. A) I, II, III, and IV B) I and II C) V only D) II, III, and IV

II, III and IV Despite being called "the basis" when the stock is withdrawn, there was no income tax liability when the stock was contributed to the plan. Of the $300,000 Lauren received as a lump-sum distribution from the stock bonus plan, $100,000 is net unrealized appreciation (NUA) and will be taxed at the long-term capital gain rate. The remaining $200,000 is taxed at Lauren's ordinary income tax rate in the year of the lump-sum distribution. Because Lauren sold the stock within six months of distribution, the $10,000 post-distribution appreciation is taxed as short-term capital gain.

In which of these circumstances is income considered to be constructively received by an employee? I. The employee's receipt of the income is subject to substantial limitations or restrictions. II. The income is credited to the employee's account. III. The income is set apart for the employee. IV. The income is made available so the employee may draw on it anytime, or could have drawn on it during the tax year if notice of intent to withdraw had been given. A) I and IV B) II, III, and IV C) II and III D) II and IV

II, III and IV Income is not constructively received if the employee's control of its receipt is subject to substantial limitations or restrictions.

Adam, age 48, and Mary, age 47, were married for 15 years when they divorced last year. Adam died this year. They have two young children, ages 10 and 12, who are cared for by Mary. Adam's 70-year-old mother, Sarah, also survived him. At the time of Adam's death, he was currently, but not fully, insured under Social Security. What benefits are Adam's survivors entitled to under the Social Security program? I. A dependent parent's benefit II. A lump-sum death benefit of $255 III. A children's benefit based on a percentage of Adam's primary insurance amount (PIA) IV. A surviving spouse benefit to take care of a dependent child A) III and IV B) I, II, and III C) II, III, and IV D) I and II

II, III and IV One lump-sum death benefit of $255 is payable if he was fully or currently insured. The children's benefit is payable because Adam was currently insured. Adam's divorced spouse (the children's mother) is entitled to a caretaker's benefit for caring for his children under age 16. Adam's mother would only be entitled to a benefit if Adam was fully insured and he had been providing at least half of her support at the time of his death.

In which of these retirement plans can forfeitures be reallocated to participants to increase account balances of plan participants? I. Traditional defined benefit plans II. Cash balance plans III. Employee stock ownership plans (ESOPs) IV. SIMPLE IRAs

III Only Statement I is incorrect because defined benefit plan forfeitures must be used to offset plan costs. Statement II is incorrect because cash balance plan forfeitures must be used to offset plan costs. A cash balance plan is a type of defined benefit pension plan. Statement III is correct; ESOPs can use forfeitures to increase the account balances of remaining participants. An ESOP is a type of profit-sharing plan featuring ownership of employer stock. Statement IV is incorrect because SIMPLE IRAs require 100% immediate vesting.

Todd, age 60, has made contributions of $75,000 to his traditional IRA, of which $15,000 were nondeductible contributions. He is considering taking a $20,000 distribution from his IRA, which currently has a fair market value of $175,000. When calculating the nontaxable portion of his IRA, which of the following formulas is CORRECT? I. Nontaxable portion = nondeductible contributions ÷ [(IRA balance at the beginning of the year + the IRA balance at the end of the year) ÷ 2] × IRA distributions II. Nontaxable portion = nondeductible contributions ÷ (IRA balance at the beginning of the year + any distributions taken during the year) × IRA distributions III. Nontaxable portion = [(nondeductible contributions prior to current year + all contributions for current year) ÷ (balances at end of current year + distributions received in current year)] × total distributions during current year A) I only B) III only C) II

III Only The correct formula is: nontaxable portion = [(nondeductible contributions prior to current year + all contributions for current year) ÷ (balances at end of current year + distributions received in current year)] × total distributions during current year

A cashless exercise of stock options: I. provides the employee with cash only. II. provides the employee with stock only. III. involves no cash outlay by the employee. IV. works best for employees with insufficient cash to exercise the option.

III and IV The answer is III and IV. The employee receives the net amount of the stock (the stock remaining after the sale of shares) or all cash. When an option is exercised, the employee has to pay to buy the shares. If the employee does not have enough cash to exercise the options, then a cashless option becomes a viable choice. The option is exercised, after which sufficient stock is immediately sold for the fair market value to realize the cash needed to pay for the exercise and any costs and taxes associated with the sale. The employee receives the net amount of the stock in either stock or cash.

Which of these are disadvantages of an excess benefit plan? I. The plan is subject to all ERISA requirements. II. Vesting schedules may not favor any class of employees.

Neither I nor II Excess benefit plans are not subject to ERISA requirements because they are unfunded plans. These plans typically favor highly compensated employees.

If nondeductible contributions have been made to an IRA, then distributions from the IRA A) would be tax free up to the amount of the nondeductible contributions. B) would be based upon first-in first-out to determine if any of the distribution would be taxable. C) are only partially taxable. D) The nondeductible portion of the IRA is treated as a Roth IRA.

A portion of these distributions is considered to be a nontaxable return of contribution.

The 401(k) for TBH, Inc. is preparing for ADP nondiscrimination testing. The nonhighly compensated employees average 7.2%. Ahmed, age 44, is the only highly compensated employee. He makes $200,000. How much can Ahmed defer into his 401(k)? A) $18,400 B) $19,450 C) $22,500 D) $18,000

$18,400 The answer is $18,400. The non-highly compensated employees average 7.2%. The ADP Test has two parts: the 1.25 times test and the 200% limited to a max of 2% test. The plan must pass at least one of these two tests, so the greater number is the actual limit on highly compensated employees. 1.25 × 7.2% = 9% 200% × 7.2 % = 14.4%. However, this test also has a 2% maximum difference. 7.2% + 2% = 9.2% 9.2% is the higher number so Ahmed is limited to 9.2%. This is $18,400 ($200,000 × .092 = $18,400).

Juan has worked for the ABC Company for 23 years. The defined benefit plan pays 2% of an employee's average of his or her high five highest years of income. Juan's average is $60,000. How much will he receive each month if he retires this year? A) $3,000/month B) $1,200/month C) $2,300/month D) $800/month

$2300/month Juan will receive $2,300 per month. $60,000 × 0.02 × 23 ÷ 12 = $2,300/month

Mark and Christine, both age 36, file their taxes jointly. Mark has been out of work for over 18 months. He earned $1,746 this year doing odd jobs. Christine earned $84,000 this year and is an active participant in the 401(k) plan at work. She contributed $2,700 to her IRA and the maximum to Mark's IRA for this year. How much can they deduct for their IRA contributions? A) $0 B) $9,200 C) $4,446 D) $6,500

$9200 The maximum contribution for spousal IRAs is $6,500 for 2023. The combined contribution for the husband and wife, however, cannot exceed the total compensation of both spouses. Also, the deduction cannot exceed the contribution. While she has plenty of earned income to cover them both, she only contributed $2,700 to her IRA. Thus, she can only deduct $2,700. Because she is an active participant you must check to see if her deduction is being phased out. In this case, their combined income is well below the start of the phaseout range for the active participant and even further below the phaseout range for the spouse who is not an active participant.

Bernie is a participant in his employer's noncontributory employee stock ownership plan (ESOP). Two years ago, his employer contributed stock with a fair market value of $30,000 into Bernie's account. Bernie retired one year later and took distribution of the stock when its fair market value was $40,000. Two years after his retirement, Bernie sold the stock for $50,000. What is the appropriate tax treatment available to Bernie upon sale of the stock? A) $20,000 long-term capital gain B) $20,000 ordinary income C) $50,000 ordinary income D) $10,000 long-term capital gain

A) $20,000 long-term capital gain Employees are not taxed on the stock in an ESOP until such time as the stock is distributed. Upon distribution, the employee must pay ordinary income taxes on the fair market value of the stock when it was contributed to the plan on his behalf. Any net unrealized appreciation (NUA) at that time can be deferred until the stock's subsequent sale. Upon the subsequent sale of the stock, the NUA portion will be treated as long-term capital gain. Additionally, the growth of the stock subsequent to the distribution will receive long-term capital gain treatment because Bernie held the stock longer than one year after distribution. Therefore, the appropriate tax treatment available to Bernie upon sale of the stock is a $20,000 long-term capital gain.

Stewart and Abby, both age 35, plan to contribute a total of $13,000 to their IRAs for this tax year. They both work outside the home, and they file a joint income tax return. Stewart is a teacher at the local high school and participates in a 403(b) plan. Abby's employer does not provide a retirement plan. They expect that their MAGI in 2023 will be $145,000. What amount, if any, can they deduct for their IRA contributions? A) $6,500 B) $13,000 C) $7,500 D) $0

A) $6,500 An individual is not denied a deduction for his IRA contribution simply because of the other spouse's active participation, unless the couple's combined AGI exceeds $228,000 (2023). Based on their AGI, Abby, who is not an active participant, will be able to deduct a contribution of up to $6,500 to an IRA. Since their combined AGI is too high for Stewart to make a deductible IRA contribution, he should consider contributing to a Roth IRA.

Which one of the following would NOT be an acceptable course of action for a 401(k) plan that has failed the actual deferral percentage (ADP) test? A) A corrective distribution of a portion of the eligible nonhighly compensated employees' deferrals B) A qualified nonelective contribution for all eligible nonhighly compensated employees C) A qualified matching contribution for all eligible nonhighly compensation employees D) A corrective distribution to the eligible highly compensated employees

A) A corrective distribution of a portion of the eligible nonhighly compensated employees' deferrals The ADP test fails when too much is being saved or contributed on behalf of highly compensated employees. In order to bring the plan into compliance it would be necessary to make a corrective distribution to a portion of the highly compensated employee deferrals, not the nonhighly compensated employee deferrals. The other three options are stated correctly.

Which of these statements regarding contributions to a traditional IRA is CORRECT? A) A taxpayer who is considered an active participant in an employer-sponsored retirement plan may also be eligible to contribute the maximum amount to a traditional IRA. B) Deductible contributions to an employed individual's IRA and an unemployed spouse's spousal IRA are limited to the lesser of 100% of earned income or $3,000. C) There are no restrictions on the deductibility of contributions, provided the individual is an active participant in an employer-sponsored retirement plan. D) Alimony and rental income are considered earned income for the purposes of making a deductible contribution to a traditional IRA.

A) A taxpayer who is considered an active participant in an employer-sponsored retirement plan may also be eligible to contribute the maximum amount to a traditional IRA.

Which of the following does NOT meet the definition of active participation in a retirement plan for the purposes of determining the deductibility of IRA contributions? A) An employee is eligible to defer salary to his employer's Section 401(k) plan. B) Kerry has benefits accrued for him under his employer's defined benefit pension plan. C) Betty has employee forfeitures reallocated to her account by her employer but makes no elective deferrals in the same plan year. D) Matt makes contributions to his employer's qualified plan.

A) An employee is eligible to defer salary to his employer's Section 401(k) plan. For the Section 401(k) retirement plan, an employee is considered covered as long as he or she is eligible to defer part of his compensation; he or she does not have to actually make contributions. Note, however, that active participation for purposes of determining deductibility of IRA contributions differs from being covered under a qualified plan for nondiscrimination testing. Specifically, an employee must be contributing to the plan, having employer contributions to the plan, forfeitures reallocated on his or her behalf, or accruing a defined benefit before he or she is considered an active participant in a qualified plan for IRA purposes. Plan earnings alone do not make someone an active participant. Also, receiving retirement plan benefits do not make someone an active participant.

Which of the following would NOT be a permitted disparity for a defined benefit plan that uses Social Security integration? A) An excess benefit percentage of 60%, if the base percentage is 30% B) An excess benefit percentage of 20%, if the base percentage is 15% C) An excess benefit percentage of 10% if the base percentage is 5% D) An excess benefit percentage of 40%, if the base percentage is 20%

A) An excess benefit percentage of 60%, if the base percentage is 30% Base percentage + permitted disparity = excess benefit percentage. The permitted disparity is the base percentage, up to a maximum of 26.25% (0.75% per year for up to 35 years).

Which of these qualification requirements applies to an employee stock ownership plan (ESOP)? A) Assets may be invested primarily in qualifying employer securities. B) Stock is sold via a public offering and the cash received on the sale is contributed to the qualified plan by the employer. C) At retirement, a lump-sum distribution of employer securities is subject to ordinary income tax on the fair market value of the securities at the time of the distribution. D) No more than 50% of plan assets may be invested in employer securities.

A) Assets may be invested primarily in qualifying employer securities. ESOP assets may be invested primarily in employer securities. This is an exception to the normal rule prohibiting more than 10% of qualified plan assets to be held in employer stock or securities. In an ESOP, stock is not sold to the public, but, rather, the plan trustee purchases stock from the employer. A lump-sum distribution of employer securities may be eligible for NUA treatment and LTCG taxation on the NUA portion, rather than ordinary income taxes on the entire distribution.

Blake and Margaret are married, file their income taxes separately, and are both age 59½. Blake makes elective deferrals into his employer's Section 401(k) plan. Margaret works for a nonprofit and makes elective deferrals to her employer's Section 403(b) plan. In 2023, Blake has a MAGI of $45,000 and Margaret has a MAGI of $40,000. Which of the following statements regarding Blake and Margaret and their retirement planning is FALSE? A) Blake and Margaret may make a deductible contribution to an IRA in 2023. B) Because Margaret participates in a Section 403(b) plan, she is considered an active participant. C) Blake and Margaret cannot make a deductible contribution to an IRA in 2023. D) Blake is an active participant.

A) Blake and Margaret may make a deductible contribution to an IRA in 2023. If both spouses are active participants in an employer-sponsored retirement plan, contributions to an IRA are phased out when married and filing separately if the individual's MAGI exceeds $10,000 (not indexed).

Maria has a traditional IRA. She named her daughter, Faith, as beneficiary of the account. Faith was disabled in an accident five years ago. If Maria dies at age 75 (when Faith was 50), which of these statements is CORRECT? A) Faith inherits the IRA as an eligible designated beneficiary. B) Faith can make new contributions to the inherited IRA after the money is transferred into the inherited account. Since she is disabled and has no earned income, the source of these new annual contributions would have to be spousal IRA contributions or money rolled into the inherited IRA from a retirement account Faith owned previously. C) Faith cannot transfer the inherited funds to an inherited IRA via a direct trustee-to-trustee transfer and name her own beneficiary. D) Because Faith is an eligible designated beneficiary (EDB), she is allowed to roll the IRA to her own name and circumstances.

A) Faith inherits the IRA as an eligible designated beneficiary. The answer is Faith inherits the IRA as an eligible designated beneficiary (EDB) because she was disabled on the day Maria died. A non-spouse beneficiary, including a non-spouse EDB, may only use a trustee-to-trustee transfer of the decedent's balance from a deceased's qualified plan, Section 403(b) plan, governmental Section 457 plan, or IRA to her own inherited IRA. What no non-spouse beneficiary can ever do is "roll" inherited money into an inherited account. Transferring retirement money into an inherited account means the first institution is verifying that the money is coming from the account of a person now deceased. If the law allowed anyone to "roll" inherited money into the new inherited account, there would be a temptation for someone to put money into an inherited IRA that was not coming from the account of a deceased person. Why? Because distributions from an inherited account are always an exception to the 10% early withdrawal penalty (EWP). So why do we allow a spouse to "roll" inherited money into their own name? Because a married couple is legally considered one person. Thus, the retirement accounts for each spouse are really for that married couple's combined retirement and thus for that generation's retirement. Also, when a surviving spouse moves inherited money into their own name, any withdrawals from the spouse's own retirement account will not automatically be an exception to the 10% EWP. Next, even though Faith is an EDB (due to being disabled on the day Maria passed away), Faith cannot roll the inherited money into her own name and circumstances. First, she is not a spouse and thus, she cannot "roll" inherited retirement money at all. Second, only a spouse can move money into their own name (as opposed to into an inherited accoun

Charlie contributed $2,000 to Roth IRA 1 last year, when he was age 24, and $2,000 to Roth IRA 2 this year. Two years from now, Roth IRA 1 will have a balance of $2,650, and Roth IRA 2 will have a balance of $2,590, and Charlie will close Roth IRA 1, receiving the balance of $2,650. Which one of the following statements best describes his tax and penalty status for that year? A) He will pay neither taxes nor a penalty. B) He cannot make any withdrawals, because the money has not been in the Roth IRA for five years or longer. C) He must pay taxes and a penalty on the full distribution. D) He only pays ordinary taxes, because Roth IRA distributions are not subject to a penalty.

A) He will pay neither taxes nor a penalty. The distribution is not qualified because Charlie is under age 59½, and he is withdrawing the money before the waiting period of five tax years. None of the withdrawal, however, is included in Charlie's taxable income because the $2,650 sum is less than the aggregate total of his contributions ($4,000). No penalty applies since the withdrawal is not taxable.

Mary is considering naming her estate as the beneficiary of her traditional IRA. Which of these is a disadvantage of this approach? A) Her estate cannot be treated as a designated beneficiary for purposes of determining the distribution period after she dies. B) The estate will probably pay no more income tax on the IRA distributions than would an individual beneficiary. C) Postmortem distributions will not go through the probate process. D) Postmortem distributions are given preferential treatment.

A) Her estate cannot be treated as a designated beneficiary for purposes of determining the distribution period after she dies. The estate cannot be treated as a designated beneficiary for purposes of determining required minimum distributions, and the estate will begin paying income tax at the maximum rate at a much lower level than an individual beneficiary would. Post-mortem distributions will go through the probate process, and distributions will lose any preferential treatment. Postmortem distributions will go through the probate process, and distributions lose any preferential treatment.

Which of the following statements regarding qualified retirement plans is CORRECT? I. Money purchase pension plans, employee stock ownership plans (ESOP), target benefit pension plans, and stock bonus plans are all examples of qualified retirement plans. II. Top-hat plans and cash balance pension plans are examples of qualified retirement plans. A) I only B) Both I and II C) Neither I nor II D) II only

A) I only Statement II is incorrect because top-hat plans are nonqualified retirement plans. A cash balance pension plan, however, is an example of a qualified plan.

Which of the following reasons for an early distribution from a qualified retirement plan is NOT an exception to the 10% penalty? A) It is a distribution for higher-education costs. B) The plan owner becomes totally and permanently disabled. C) It is made after separation from service from an employer-sponsor of the plan after age 55. D) The distribution is made to a beneficiary of the account due to the owner's death.

A) It is a distribution for higher-education costs. The exception from the 10% early distribution penalty for distributions for higher-education costs only applies to IRA distributions.

Which of the following groups would NOT benefit from using Roth IRAs? A) Low-income wage earners who need current deductions B) High-income wage earners who have exhausted the tax benefits of other tax-favored vehicles C) Low-income wage earners who are active participants in their employers' plans and are not eligible to make deductible IRA contributions D) Taxpayers who anticipate being in a higher tax bracket in their retirement years

A) Low-income wage earners who need current deductions Roth IRAs offer no current deductions. Low-income wage earners needing current deductions are better served by traditional IRAs.

Mark attained age 73 this year. He does not plan to retire from his position with Big Trucks, Inc., until his birthday on December 1, when he is 76. Mark is a 3% shareholder in Big Trucks. When must Mark begin to receive required minimum distributions (RMDs) from his qualified retirement plan at Big Trucks? A) Mark is not required to begin his RMDs until April 1 of the year following his actual retirement from Big Trucks. B) Mark is not required to receive his first RMD until December 31 of the year following his actual retirement date from Big Trucks. C) Because Mark is a shareholder in Big Trucks, he must receive his first RMD by April 1 of next year. D) Because Mark is still employed by Big Trucks, he is not required to take his first RMD until December 31 of the year he actually retires from Big Trucks.

A) Mark is not required to begin his RMDs until April 1 of the year following his actual retirement from Big Trucks. Participants in qualified plans, Section 403(b) plans, and governmental Section 457 plans may defer the required beginning date until April 1 following the year of retirement, if the participant continues to work after attaining age 73. If the employee-participant owns more than 5% of the business sponsoring the retirement plan, the RMD may not be deferred but must be taken by April 1 of the year after the employee attains age 73. SECURE 2.0 raised the normal trigger year for RMDs from 72 to 73 for 2023 and years following.

The Acme Corporation has six owners, ranging in age from 30 to 60 years old, and 25 rank-and-file employees. The owners want to adopt a qualified retirement plan that will allow them to maximize the contributions to the owners' accounts and to minimize the contributions to the accounts of the rank-and- file employees. Which of the following plans would best meet the owners' needs? A) New comparability plan B) Section 401(k) plan C) Age-based profit-sharing plan D) Keogh plan

A) New comparability plan A new comparability plan would allow the owners to divide the participants into two classes based on their compensation levels, and to allocate different contribution levels to the classes. An age-based profit-sharing plan wouldn't meet their objectives because the owners' ages are significantly different. Section 401(k) plans are subject to discrimination testing, and a Keogh plan is inappropriate because the owners are not self-employed.

Which of the following statements is NOT a requirement for the beneficiaries of a trust to be treated as a designated beneficiary of a qualified plan or an IRA? A) The beneficiary of the trust is named on the decedent's retirement account as a named beneficiary. B) The trust is valid under state law. C) The trust is irrevocable at the participant's death. D) The appropriate documentation is provided to the plan administrator.

A) The beneficiary of the trust is named on the decedent's retirement account as a named beneficiary. The beneficiaries of the trust must be named (identified) in the trust instrument. Only the trust needs to be named as a beneficiary on the account.

Which of the following retirement plans, maintained by an eligible employer, would also permit the employer to establish a SIMPLE IRA? A) Union plan bargained in good faith B) Money purchase pension plan C) Traditional Section 401(k) plan D) Section 403(b) plan

A) Union plan bargained in good faith To establish a SIMPLE (IRA or Section 401(k)), an employer cannot generally maintain another retirement plan. However, a plan bargained in good faith with a union is allowed because Congress is more worried about powerful businessowners using a retirement plan in a discriminatory manner.

Which of these is CORRECT about a Roth IRA? A) Withdrawals of earnings up to $10,000 from a Roth IRA for the purchase of a first home can be penalty free if the five-year holding period has been met. B) An individual can contribute $6,500 annually to a regular IRA and also an additional $6,500 annually to a Roth IRA in 2023. C) If a nonqualifying distribution is made before age 59½, the principal is subject to the 10% penalty, but it is not considered to be taxable income. D) As with conventional IRAs, distributions from a Roth IRA must begin by April 1 of the year following the year the participant reaches age 73.

A) Withdrawals of earnings up to $10,000 from a Roth IRA for the purchase of a first home can be penalty free if the five-year holding period has been met. Withdrawals of earnings from a Roth IRA are not penalized under these circumstances if the five-year holding period has been met.

All of the following are considerations for converting distributions from qualified plans or a traditional IRA to a Roth IRA except: A) the Roth IRA conversion is more appropriate when the income tax rate is lower at the time of distribution than at the time of conversion. B) any portion of an IRA can be converted to a Roth IRA. C) the Roth IRA conversion becomes more appropriate the longer the period of distributions. D) one advantage of a conversion to the Roth IRA is that the Roth IRA will not be subject to required minimum distributions (RMD) during the life of the original owner.

A) the Roth IRA conversion is more appropriate when the income tax rate is lower at the time of distribution than at the time of conversion. The answer is the Roth IRA conversion is more appropriate when the income tax rate is lower at the time of distribution than at the time of conversion. The Roth IRA conversion is more appropriate when the income tax rate is the same or higher at the time of distribution than at the time of conversion.

All of the following statements regarding target benefit pension plans are correct except A) the plans are covered by Pension Benefit Guaranty Corporation (PBGC) insurance. B) minimum funding standards apply. C) older participants are favored in a target benefit pension plan. D) each employee has an individual account.

A) the plans are covered by Pension Benefit Guaranty Corporation (PBGC) insurance. Target benefit pension plans are defined contribution plans, so they are not covered by PBGC insurance, and employees have their own individual accounts. Because they are pension plans, minimum funding standards apply. Similar to defined benefit plans, target benefit pension plans favor older participants with a larger contribution at plan entry than a younger person joining the firm with the same compensation.

Dorinda, age 45, was divorced in 2017. Due to an inheritance, she finds she doesn't really need the money she gets from her ex-spouse as part of the divorce agreement. She receives $40,000 of child support and $3,600 of alimony in 2023. She wants to save the alimony for her retirement years. What is the amount that Dorinda can contribute to a traditional IRA? A) $6,500 B) $2,400 C) $3,600 D) $0

Alimony from pre-2019 divorces is considered earned income for the purpose of making a contribution to an IRA. Child support is not considered earned income for IRA contribution purposes. Because this is the only income she receives that is considered earned income, she may make a contribution of 100% of earned income up to a maximum contribution of the lesser of $6,500 (2023) to a traditional IRA, or $3,600, the amount of her alimony. Note: Divorces finalized during and after 2019 fall under the rules that will not include alimony as tax deductible to the payer and taxable to the receiver. As such, this will not be considered earned income. This divorce happened prior to this Tax Reform Act and thus follows the old rules.

Ryan wants to take a distribution from his SIMPLE 401(k) account balance from his previous employer and move it in an IRA at his local banking institution. Which of these statements regarding this situation is CORRECT? A) A SIMPLE 401(k) plan may include a two-to-six year graded vesting schedule. B) A direct transfer from Ryan's SIMPLE 401(k) to an IRA is not subject to the mandatory 20% income tax withholding requirement. C) A distribution from the SIMPLE 401(k) plan is not subject to mandatory 20% withholding. D) Ryan cannot transfer employer matching contributions for at least five years.

B) A direct transfer from Ryan's SIMPLE 401(k) to an IRA is not subject to the mandatory 20% income tax withholding requirement. SIMPLE 401(k) plans are qualified plans and are subject to mandatory 20% income tax withholding for a distribution that is not a direct trustee-to-trustee transfer or a direct rollover (in which the check is made out to the next retirement plan custodian for the benefit of the person, but the check is mailed to the ex-participant to be forwarded to the new custodian). When there is a direct transfer of a distribution from a qualified plan to an IRA, the mandatory 20% withholding rule does not apply. Vesting is always immediate for employer contributions to a SIMPLE plan.

Which of these statements regarding the basic provisions of tax-sheltered annuities (TSAs)/Section 403(b) plans is FALSE? A) TSAs are available to all eligible employees of Section 501(c)(3) organizations who adopt such a plan. B) A special catch-up provision for eligible Section 403(b) participants allows up to a $45,000 (2 × $22,500 in 2023) elective deferral in the last 3 years of employment before retirement. C) An eligible employee may be able to use both a special catch-up provision and an over-age-50 catch-up provision in the same year. D) If an employee has at least 15 years of service with an eligible employer, an additional catch-up contribution may be permitted.

B) A special catch-up provision for eligible Section 403(b) participants allows up to a $45,000 (2 × $22,500 in 2023) elective deferral in the last 3 years of employment before retirement. The special catch-up provision for eligible Section 403(b) participants allows a maximum additional $3,000 per year elective deferral until the $15,000 lifetime maximum catch-up is reached. The ability to double the worker contribution in the last three years before the retirement plan document's normal retirement age applies to governmental 457 plans.

Fernando, age 45, participates in his employer's defined benefit pension plan. This plan provides for a retirement benefit of 2% of earnings for each year of service with the company and, given Fernando's projected service of 20 years, will provide him with a benefit of 40% of final average pay at age 65. What type of benefit formula is this plan using? A) A flat benefit formula B) A unit benefit formula C) A discretionary formula D) A flat percentage of earnings formula

B) A unit benefit formula Because this formula considers both compensation and years of service, the defined benefit plan is using a unit benefit formula. This type of formula is frequently used to reward continued service with the same employer.

Which of the following does NOT meet the definition of active participation in a retirement plan for the purposes of determining the deductibility of IRA contributions? A) Matt makes contributions to his employer's qualified plan. B) An employee is eligible to defer salary to his employer's Section 401(k) plan. C) Betty has employee forfeitures reallocated to her account by her employer but makes no elective deferrals in the same plan year. D) Kerry has benefits accrued for him under his employer's defined benefit pension plan.

B) An employee is eligible to defer salary to his employer's Section 401(k) plan. For the Section 401(k) retirement plan, an employee is considered covered as long as he or she is eligible to defer part of his compensation; he or she does not have to actually make contributions. Note, however, that active participation for purposes of determining deductibility of IRA contributions differs from being covered under a qualified plan for nondiscrimination testing. Specifically, an employee must be contributing to the plan, having employer contributions to the plan, forfeitures reallocated on his or her behalf, or accruing a defined benefit before he or she is considered an active participant in a qualified plan for IRA purposes. Plan earnings alone do not make someone an active participant. Also, receiving retirement plan benefits do not make someone an active participant.

Jane Paschal, age 45. has contributed $1,000 each year to a Roth IRA, beginning with an initial payment of $500 on December 31st five years ago. She wants to know when the soonest she could begin making qualified distributions. Which one of the following statements represent what you should tell her? A) After December 31st of last year, the five years will have elapsed, and the absolute earliest she could begin making qualified distributions would when she reaches age 59½. B) Any distribution she takes after January 1st this year, will meet the five-year requirement. C) Any distributions for medical expenses in excess of 7.5% would qualify as a tax-free distribution after satisfying the five-year holding period even if she has not attained age 59½. D) Qualified higher education expenses are one of the reasons a withdrawal may be a qualified distribution.

B) Any distribution she takes after January 1st this year, will meet the five-year requirement. The clock started on January 1st five years ago, so five years will have elapsed on January 1s this year (meaning after December 31st of last year). A Roth IRA owner is required to hold the account for a minimum of five years to qualify for tax-free distributions. In addition, the owner must be at least age 59½, dead, disabled, or withdrawing up to $10,000 of Roth IRA earnings for qualified first-time homebuyer expenses. Notice that higher education expenses are NOT a reason for a qualified distribution (even though they are an exception to the 10% EWP for IRAs, but not employer Roth accounts)

Which statements regarding TSAs and Section 457 plans are CORRECT? A) Of these two plans, only the 403(b) must meet the minimum distributions requirements that apply to qualified plans. B) Both plans may be funded entirely by participant contributions. C) Both plans allow net unrealized appreciation tax treatment for lump-sum distributions. D) Participation in either a TSA or a Section 457 plan will cause an individual to be considered an active participant for purposes of phasing out the deductibility of traditional IRA contributions.

B) Both plans may be funded entirely by participant contribution While an employer is allowed to contribute to either plan, both plans may be funded entirely by participant contributions. A Section 457 plan is a deferred compensation arrangement that will not cause a participant to be considered an active participant. Net unrealized appreciation (NUA) tax treatment is not permitted for distributions from either plan. Finally, both plans required RMDs.

Which of these is a type of defined contribution profit-sharing plan? A) Target benefit pension plan B) Employee stock ownership plan (ESOP) C) Cash balance pension plan D) Money purchase pension plan

B) Employee stock ownership plan (ESOP) A cash balance pension plan is a type of defined benefit pension plan. An ESOP is a defined contribution profit-sharing plan.

In a money purchase pension plan that utilizes plan forfeitures to reduce future employer plan contributions, which of the following components must be factored into the calculation of the maximum annual addition limit? (CFP® Certification Examination, released 11/94) I. Forfeitures that otherwise would have been reallocated II. Annual earnings on all employer and employee contributions III. Rollover contributions for the year IV. Employer and employee contributions to all defined contribution plans A) I and III B) IV only C) I, II, III, and IV D) I, II, and III

B) IV only Because the forfeitures will be used to reduce future employer contributions, they will not count against the annual additions limit. Defined contribution plans do not consider earnings on investments when calculating contributions.

Jane, 53, has retired and taken a full distribution from her employee stock ownership plan (ESOP). Over the years, her employer made a total of $40,000 in contributions into the plan for her, and the stock is currently valued at $110,000. Which one of the following statements best describes the tax implications of the distribution that Jane has taken? A) Jane will owe long-term capital gain taxes on $70,000, which she must pay this year because this is the year of the distribution. B) Jane will owe ordinary income taxes and a 10% penalty tax on $40,000. C) Jane will owe just ordinary income taxes on $40,000. D) Jane will owe ordinary income tax on the entire $110,000 distribution from the ESOP.

B) Jane will owe ordinary income taxes and a 10% penalty tax on $40,000. Because Jane was not at least age 55 at the end of the year when she separated from service, she will owe not only ordinary income taxes but also a 10% early withdrawal penalty tax on the $40,000 basis. She will be taxed at the long-term capital gains rate for the $70,000 of NUA, but she is not subject to this tax until she actually sells the stock. When the stock is eventually sold, the $70,000 NUA amount will usually be a long-term capital gain. If the stock is eventually sold for a profit above $110,000, the gain over the $110,000 mark is long- or short-term capital gain depending how long after the distribution the shares are sold. If the stock is eventually sold for less than $110,000, the loss from $110,000 decreases the NUA amount and thus lowers the long-term capital gain. If Jane dies while owning the stock, the NUA amount will not get a stepped-up basis, but the rest of the stock's value on the date of death will get a stepped-up basis. Thus, the easiest way to determine the stepped-up basis is to subtract the NUA from the fair market value of the stock on the date of death.

Kiersten, a single 44-year-old, works for two unrelated employers: Acme, Inc., and Douglas School District. With Acme, she qualifies for a 401(k) plan; she also qualifies for the Douglas School District Section 457 plan. She earns $15,000 from Acme, Inc., and $26,000 from Douglas School District. She received a large inheritance and would like to save as much as possible for her retirement through her employers. Which one of the following is CORRECT in stating the amounts that Kiersten could contribute to the two plans in 2023? A) Kiersten could only contribute a total of $22,500 in both plans, but would be limited to $15,000 in the Acme 401(k) plan. If she contributes $15,000 to the Acme 401(k) plan, she could only put $7,500 into the Douglas School District 457 plan. B) Kiersten could contribute up to $15,000 in the Acme 401(k) plan and $22,500 in the Douglas School District 457 plan. C) Kiersten could contribute

B) Kiersten could contribute up to $15,000 in the Acme 401(k) plan and $22,500 in the Douglas School District 457 plan. The basic limit on elective deferrals with both a 401(k) plan and 457 plan is $22,500 in 2023, or 100% of the employee's compensation, whichever is less. In this case, she only makes $15,000 with Acme, so that is the limit for Acme's 401(k). The 457 plan is not aggregated with other deferral plans. No one can ever do both the age 50 catch-up and also the double the basis limit to a 457 plan.

Which of the following is CORRECT in relation to a nonqualified distribution from a Roth IRA? A) A nonqualified distribution from a Roth IRA held for five years will not be subject to ordinary income tax or an early withdrawal penalty because the holding period requirement has been satisfied. B) Regular contributions will be deemed the first distributed and will never be subject to income tax or an early withdrawal penalty. C) A nonqualified distribution attributable to a conversion within five years will be subject to ordinary income tax and an early withdrawal penalty. D) A nonqualified distribution will always incur a penalty.

B) Regular contributions will be deemed the first distributed and will never be subject to income tax or an early withdrawal penalty. A distribution from a Roth IRA, qualified or nonqualified, will be attributed as first coming from regular contributions, and that amount will never be subject to income tax or an early withdrawal penalty. A nonqualified distribution will only be subject to penalty if the distribution is attributable to earnings or to a conversion within five years. A distribution attributed to a conversion within five years is subject to an early withdrawal penalty, but is not subject to regular income tax. A distribution from a Roth IRA must meet both the five year-holding period requirement and one of the other qualified circumstances (death, disability, first-time home purchase, age 59½).

Blake, age 75, is required to take substantial required minimum distributions (RMDs) from his qualified retirement plan. He has no current need for the cash and has established traditional IRAs with his children as beneficiaries and wishes to deposit the RMD distributions in equal amounts to each IRA within 60 days of the distributions to eventually benefit his children. Which of these statements regarding Blake's rollover of the RMDs is CORRECT? A) Because Blake is over age 73, he may not roll over the RMDs to a traditional IRA, but he may roll over the RMDs to a Roth IRA. B) Required minimum distributions may not be rolled over. C) Required minimum distributions may not be rolled over, but Blake may make equivalent contributions to the traditional IRAs under any circumstance as long as the contributions are within 60 days of receiving his RMDs. D) A good plan is for Blake to roll over the distribution within 60 da

B) Required minimum distributions may not be rolled over. Blake may not roll over the RMD distributions, but if he has earned income, he may make contributions to a traditional or Roth IRA. The new contributions must be based on earned income and be subject to the normal annual limits and contribution timing rules.

Margaret is a 29-year-old attorney with her own law practice. She has hired four part-time employees over the past five years to assist her. Each of these employees works approximately 200 hours per year, earning an average annual salary of $4,000. Margaret would like to establish a retirement plan that would allow her to begin saving for her own retirement, with little administrative costs. Which one of the following plans would be most appropriate for Margaret in 2023? A) Section 457 plan B) SIMPLE IRA C) SEP plan D) Traditional Section 401(k) plan

B) SIMPLE IRA A SIMPLE IRA would be the most appropriate plan because it involves little administrative costs and would meet Margaret's retirement plan goals. Notice the workers are making more than $750 and less than $5,000. If a SEP would be chosen, she would have to contribute for employees who made more than $750; are at least age 21; and who have worked for her for three of the preceding five years. In other words, if a test question has part-time workers making more than the amount required to contribute to a SEP (which is indexed) but less than $5,000 (which is not indexed), then the SEP is eliminated and the answer will probably be a SIMPLE. Why? Because owners are unlikely to contribute to the retirement plan of a part-time worker. Choosing a SIMPLE always eliminates those making less than $5,000. However, a contribution must be made for part-time workers who are at least age 21, have worked there for at least three of the last five years, and make more than the very low SEP annual amount. Another separator between SEPs and SIMPLEs is that a SEP can only have an employer contribution, but SIMPLEs allow workers to save for their retirement through the plan.

Which of the following retirement plans, maintained by an employer, would also permit an eligible employer to establish a SIMPLE? A) SEP plan B) Section 457 plan C) Section 403(b) plan D) Section 401(k) plan

B) Section 457 plan To establish a SIMPLE, an employer cannot maintain another plan. However, a Section 457 plan is a nonqualified deferred compensation plan; therefore, it does not constitute a plan for purposes of establishing a SIMPLE.

A client's employer has recently implemented a traditional Section 401(k) plan as part of its profit-sharing plan. Which of these is CORRECT regarding the client's participation in the plan? A) The client will not pay Social Security (FICA) taxes on amounts paid into the plan. B) The client is immediately vested in all elective deferrals and their accrued earnings. C) The client will not pay current federal income taxes on amounts distributed from the plan. D) The client is always immediately vested in all employer-matching contributions and their accrued benefits.

B) The client is immediately vested in all elective deferrals and their accrued earnings. Because a Section 401(k) plan is a qualified defined contribution plan, the employee is immediately vested (100%) in all elective deferrals and their accrued benefits. Such deferrals are, however, subject to FICA taxes. Vesting schedules may be used with employer-matching contributions.

Teresa, 37, recently terminated her employment with Applied Dynamicals Inc. The company had no qualified plan, but she has a Roth IRA created by rolling over $25,800 from her traditional IRA 20 months ago. Her Roth account has a balance of $30,000. As a result of her layoff, she needed some cash, and so she requested that her mutual fund company send her $5,500 by check, made in her name. Which one of the following statements best describes the distribution requirements that will directly affect the amount she receives on her distribution check? A) The 20% mandatory withholding requirement will apply. B) The distribution will be income tax free because it is less than her conversion contribution. C) The distribution will be income tax free and without any premature distribution penalty because it is less than her conversion contribution. D) Teresa will owe income tax and the 10% early withdrawal penalty on the $5,50

B) The distribution will be income tax free because it is less than her conversion contribution. In this case, the mandatory withholding requirement will not apply because the payment is made from an IRA, not an employer retirement plan. The distribution will be income tax free because it is less than her conversion contribution and she paid the tax at the time of conversion. However, she will be assessed a 10% early withdrawal penalty because the distribution is from a converted IRA, five years has not elapsed, and the withdrawal does not meet any of the exceptions to the early withdrawal penalty from an IRA.

Which of these statements is CORRECT in describing the effects of actuarial methods or assumptions on contributions to a defined benefit pension plan? A) The higher the interest rate assumed by the actuary, the higher the required employer contribution for the year. B) The higher the turnover rate assumed by the actuary, the lower the required employer contribution for the year. C) Using salary scales tends to reduce the cost for funding younger employees' benefits. D) The lower the projected investment earnings on plan assets over the life of the plan, the lower the required employer contribution for the year.

B) The higher the turnover rate assumed by the actuary, the lower the required employer contribution for the year. Because defined benefit plans must apply forfeitures to reduce the employer contribution, an actuarial assumption of high turnover in a plan year will result in a lower required employer contribution.

The qualified retirement plan maintained by the ABC Corporation imposes a two-year waiting period before new employees may enter the plan. Which of the following statements is CORRECT? A) The plan is a Section 401(k) plan. B) The plan must provide 100% vesting to all employees immediately upon entry. C) The plan is in violation of ERISA. D) The plan may use the normal qualified plan vesting schedules.

B) The plan must provide 100% vesting to all employees immediately upon entry. Most types of qualified plans are allowed to increase the waiting period to two years of service, but plans adopting this rule must provide 100% vesting to all employees immediately upon entry. Section 401(k) plans cannot require a two-year waiting period.

Jill has decided to offer a retirement plan to her employees. She wants to implement a SIMPLE and is trying to decide between a SIMPLE IRA and a SIMPLE 401(k). All of the following statements apply to both types of savings incentive match plan for employees (SIMPLE) plans except A) SIMPLEs are not subject to the top-heavy rules that apply to qualified plans. B) there is a 25% penalty for early distributions from a participant's account within two years of entry into the plan. C) employer contributions are not subject to payroll taxes (FICA and FUTA). D) SIMPLE assets may only be rolled over into another SIMPLE within the first two years of initial participation in the plan by a participant.

B) there is a 25% penalty for early distributions from a participant's account within two years of entry into the plan. Only early distributions from a SIMPLE IRA within the first two years of initial participation in the plan are subject to the 25% early withdrawal penalty tax.

James, a pilot, founded an airline, Margaritaville Airways, as a sole proprietorship 15 years ago. Six years ago, he hired six employees. Now the business has grown, and he decides to incorporate. The new successor entity, Margaritaville Airways, Inc., offers a defined benefit plan. If a unit benefit formula is used, James wants to know if employees will receive credit for past years of service. Which of the following is CORRECT? A) The owner will receive credit for 10 years of service. The employees will receive no credit. B) All employees will receive credit for 10 years of past service. C) All employees will receive credit for 5 years of past service. D) Neither the owner nor the employees will receive any credit for past service.

C) All employees will receive credit for 5 years of past service. Under IRS safe harbor rules, a successor entity may recognize up to five years of service when establishing a defined benefit plan. The plan may not discriminate in favor of the highly compensated employee-owner(s).

Which of these statements regarding Section 403(b) plans are CORRECT? I. Section 403(b) plans must comply with many of the same reporting and auditing requirements that apply to Section 401(k) plans. II. Certain eligible participants in a Section 403(b) plan may defer as much as much as $33,000 into the plan in 2023. III. Section 403(b) plans may provide for plan loans to participants. IV. Funding for Section 403(b) plans is limited to mutual funds and annuities. A) I, II, and III B) I and II C) I, II, III, and IV D) III and IV

C) I, II, III, and IV All of the statements are correct. The $33,000 maximum elective deferral for 2023 includes the $22,500 basic limit; the extra $3,000 per year for up to $15,000 lifetime for employees of not-for-profit health care, education, and church employers; and the $7,500 catch-up for those age 50 and older.

Which of these are basic provisions of an IRC Section 401(k) plan? I. Employee elective deferrals are exempt from income tax withholding and FICA and FUTA taxes. II. An employer's deduction for contributions to a Section 401(k) plan cannot exceed 25% of covered payroll, which is not reduced by the employees' elective deferrals. III. A Section 401(k) plan cannot require as a condition of participation that an employee complete a period of service longer than one year. IV. Employee elective deferrals may be made from salary or bonuses. A) I and III B) II and IV C) II, III, and IV D) I and IV

C) II, III, and IV Options II, III, and IV correctly describe the 25% employer deduction limitation, eligibility requirement, and potential sources of employee elective deferrals for Section 401(k) plans. Option I is incorrect because employee elective deferrals (i.e., salary deferrals) are subject to FICA and FUTA taxes. Option III is correct. There can be a two-year period of service requirement if the participants are 100% immediately vested for some other employer retirement plans. However, this does not apply for 401(k) plans.

n increase in which of these factors will increase plan costs for a defined benefit pension plan? A) Investment returns B) Mortality C) Life expectancies D) Employee turnover

C) Life expectancies An increase in employee life expectancies will increase the potential costs of the plan, so life expectancies have a direct impact on plan costs.

Which of the following is NOT an option unique to a surviving spouse when someone dies with a retirement account or IRA? A) Only surviving spouses can recalculate their life expectancy each year. B) Only surviving spouses can absorb the money into their own name and treat the money as if the decedent never existed. C) Only surviving spouses can extend RMDs longer than 10 years. D) When the decedent dies prior to the required beginning date, only a surviving spouse can delay RMDs until the decedent would have been 73.

C) Only surviving spouses can extend RMDs longer than 10 years. Eligible designated beneficiaries (EDBs) can extend RMDs past 10 years by taking the distribution according to the Table I life expectancy. For a non-spouse EDB, 1 is subtracted from the previous year's life expectancy factor. Spouses, however, return to Table I each and every year to determine the life expectancy factor. This is a great advantage because it lowers the RMDs each year compared to the "Minus 1" method.

Tom, age 54, is the sole proprietor of a small business. He is interested in adopting a retirement plan for the business. His primary goals are to make large contributions to his own retirement account and to minimize the expense and paperwork associated with the plan. Which of the following retirement plans would you recommend for Tom's business? (He makes $50,000 of self-employment income and has three part-time employees who earn $15,000 each.) A) SEP plan B) Traditional defined benefit pension plan C) SIMPLE IRA D) Section 401(k) plan funded by employee elective deferrals

C) SIMPLE IRA

Which one of these requirements is a possible disadvantage of a simplified employee pension (SEP) for an employer? A) Employer contributions to a SEP are subject to payroll taxes. B) A SEP must have a fixed contribution formula that is nondiscriminatory. C) The vesting requirements for a SEP prohibit forfeitures. D) Roth tax treatment is not available for a SEP.

C) The vesting requirements for a SEP prohibit forfeitures. SEP contributions must be 100% vested (i.e., nonforfeitable). SEPs consist of individual IRAs; there is no trustee for a SEP plan. The contribution formula of a SEP is not required to be fixed. Employer contributions to a SEP are not subject to payroll taxes. SECURE 2.0 allows a worker to elect Roth tax treatment for SEPs and SIMPLEs (SIMPLE IRAs and SIMPLE 401[k]s) as of 2023.

Jerry and Barbara recently filed for divorce after 25 years of marriage. The property settlement approved by the court included an award to Barbara of half of Jerry's vested benefit in his defined benefit pension plan. This was done via the drafting and implementation of a qualified domestic relations order (QDRO). Which of the following is an implication of the QDRO for Jerry and Barbara? A) Barbara's benefit upon receipt is not subject to income tax. B) Barbara's benefit is subject to an additional 10% penalty if received before her age 59½. C) When Jerry retires, Barbara's benefit is taxable to her. D) Under QDRO rules, Barbara is not eligible to roll over the distribution to an IRA.

C) When Jerry retires, Barbara's benefit is taxable to her. A distribution by a qualified retirement plan to an alternate payee who is a spouse or former spouse of the participant is taxable to the spouse, if it is made pursuant to a QDRO. The distribution to Barbara pursuant to a QDRO is not subject to the 10% penalty for early distribution. Barbara may roll over the distribution to an IRA.

Charles and Lucy Brown each had $100,000 in their respective IRAs on December 31 of last year. Each has named the other as beneficiary. They need to determine the amount each must withdraw once withdrawals are required. This year, Charles turned age 73 on January 2 and Lucy turned age 73 on August 4. What is the required minimum distribution (RMD) for each? (Assume the IRS RMD Joint Life Table expected return for two individuals age 73 is 19.1. The Uniform Table factor is 27.4 at age 72 and 26.5 at age 73.) A) $4,854 B) $3,650 C) $5,284 D) $3,774

D) $3,774 $100,000 ÷ 26.5 = $3,774 (The first distribution year results in the same amount for each.)

An employer-sponsored money purchase pension plan, integrated with Social Security, uses a base contribution formula of 10% for all participants and the Social Security taxable wage base as the integration level. Given this information, what is the maximum permitted excess contribution percentage? A) 20.0% B) 5.7% C) 10.0% D) 15.7%

D) 15.7% The permitted disparity rules for defined contribution plans specify that the maximum excess percentage cannot exceed the lesser of (1) two times the base contribution percentage or (2) the base contribution percentage plus 5.7%. Therefore, the excess contribution percentage is 15.7% (10% + 5.7%).

Baxter and Smith is a law firm with a defined benefit pension plan. When would the plan be required to be covered by the Pension Benefit Guaranty Corporation (PBGC)? A) If the firm employs 26 or more employees B) If the firm employs 25 or fewer employees C) If the firm employs 10 or more employees D) A professional service employer is not required to be covered by PBGC

Defined benefit plans maintained by certain professional service employers with 26 or more employees must be covered by the PBGC.

Assume a company's goal is to maximize retirement benefits to the highly compensated employees, who also happen to be the oldest employees (LO 2.3.1). Which of the following best accomplishes this goal if the company is installing a new plan? A) A target benefit pension plan B) An age-weighted profit-sharing plan C) A money purchase pension plan D) A defined benefit pension plan

D) A defined benefit pension plan The defined benefit pension plan favors older participants and generally allows larger contributions than other plans. Age-based profit-sharing plans and target benefit pension plans also favor older participants. However, age-based profit-sharing plans and target benefit pension plans are both defined contribution plans that are subject to the annual additions limit (for 2023 a maximum contribution of no more than the lesser of 100% of an employee's compensation or $66,000).

Which of these beneficiaries is entitled to move a post-death distribution from a qualified plan into an IRA? A) The oldest surviving child of the participant B) he surviving spouse of the participant C) The surviving mother of the participant D) All three choices are correct

D) All three choices are correct A spouse beneficiary can roll the distribution over into an IRA and treat it as the spouse's own; a nonspouse beneficiary can use a direct trustee-to-trustee transfer of the distribution into a specially titled inherited IRA. Our industry can use the terms "rollover" and "roll" loosely to describe moving money from one retirement account to another. If an answer on the CFP Final is correct except it uses the terms "rollover" or "roll" to describe a nonspouse moving retirement money into an inherited account after the death of the original owner, and all other answers are wrong, pick the answer that says inherited retirement money can be "rolled" into an inherited IRA. The intent of the question was to test the rules for moving inherited retirement money.

Harry, age 34, contributed $2,000 to a Roth IRA six years ago. By this year, the investments in his account had grown to $3,785. Finding himself in a financial bind, Harry is now compelled to withdraw $2,000 from this Roth IRA. What is the tax and penalty status of this withdrawal? A) Harry must pay tax and a $200 penalty. B) Harry must pay tax on the $2,000, but there is no penalty. C) This would be a prohibited transaction. D) Harry does not have to pay any tax or penalty on the $2,000 distribution, even though he is only age 34.

D) Harry does not have to pay any tax or penalty on the $2,000 distribution, even though he is only age 34. All Roth IRA contributions are made with after-tax funds, and contributions are considered to be withdrawn first, tax free, then earnings. Also, the IRC rules allow the aggregation of all Roth IRAs for this calculation. Penalties would apply only to taxable income.

Which of these is CORRECT about how the performance of a given variable will impact the employer costs and thus the contribution to a defined benefit plan? A) Life expectancy and plan costs have an inverse relationship. B) If the average age of new employees increases, plan contributions will decrease. C) If worker turnover is greater than the plan assumes, contributions will increase. D) If mortality is greater than expected, the employer contribution will decrease.

D) If mortality is greater than expected, the employer contribution will decrease. Mortality is the rate at which people pass away. If the actual mortality rate is higher than expected, plan contributions will decrease. Life expectancy and defined benefit plan contributions move in the same directions. If people live longer, the plan costs more. If new workers are older than the plan assumptions, then contributions must increase because there is less time to fund the plan. Higher turnover would mean higher reallocated forfeitures and thus lower costs. Also, people leaving an employer means their future benefits do not need to be funded.

In considering whether to convert a traditional IRA to the Roth IRA form, which of the following is a valid consideration? A) It is generally advantageous if the converted assets will remain in the Roth IRA for a relatively short time period before withdrawal. B) If the taxpayer anticipates being in a lower tax bracket at date of distribution from the Roth IRA, it generally makes sense to convert. C) If the taxpayer files as married filing separately and thereby splits income, it generally makes sense to convert. D) If the source of payment for taxes due upon conversion comes from an outside source, it generally is advantageous to convert.

D) If the source of payment for taxes due upon conversion comes from an outside source, it generally is advantageous to convert. The Roth IRA yields greater after-tax benefits than a traditional deductible IRA if the front-end tax due upon conversion is paid from funds outside the Roth IRA and an equivalent amount of funds is, thereby, available for investment.

ina is a participant in her company's employee stock ownership plan (ESOP). The company transfers 1,000 shares of employer stock at $5 per share to her account. Several years later, when the stock is $11 per share, Tina retires at age 61. If she elects to receive the stock in a lump sum at retirement and later sells the stock for $12,000, what are the tax consequences to Tina? A) Tina will have a capital gain of $1,000. B) Tina will have ordinary income of $12,000 when she sells the stock. C) Tina will be taxed on $11,000 of ordinary income when she receives the stock at retirement. D) Tina will have a capital gain of $7,000.

D) Tina will have a capital gain of $7,000. Because of the net unrealized appreciation rule, at distribution, Tina will be taxed only on $5,000, the original cost of the stock when it was contributed to the plan. Tina was taxed on the $5,000 basis in the stock when she received her distribution. Therefore, her capital gain will be $7,000 if she sells the stock for $12,000.

Which of these statements regarding simplified employee pension (SEP) plans is CORRECT? A) Contributions must be made for all employees who have attained age 21 and who have performed services for the employer for at least two of the last five years. B) Workers can decide how much they will contribute to a SEP. C) SEPs can offer retirement plan loans. D) Workers are allowed to have employer SEP contributions treated as Roth contributions.

D) Workers are allowed to have employer SEP contributions treated as Roth contributions.

Which of the following statements regarding both a SEP plan and a qualified profit-sharing plan is CORRECT? A) Annual employer contributions are mandatory. B) The investment risk is borne by the employer. C) Immediate 100% vesting of all contributions is required. D) he employer's tax-deductible contributions are limited to 25% of aggregate covered compensation.

D) he employer's tax-deductible contributions are limited to 25% of aggregate covered compensation. SEP plans and qualified profit-sharing plans are both subject to 25% of the aggregate covered compensation limit on annual employer deductions. Neither plan requires an annual mandatory employer contributions, nor is investment risk borne by the employer. Immediate 100% vesting is required in SEP plans but not in profit-sharing plans.

Marian, age 62, converted $30,000 from a traditional IRA to a Roth IRA seven years ago. Last year she converted another traditional IRA with a fair market value of $35,000 to a Roth IRA. She makes no other IRA contributions. This year she took a $40,000 distribution from her Roth IRA. This distribution is treated as $30,000 from the seven year old conversion contribution and $10,000 from last year's conversion contribution, both of which were includable in her gross income when converted. As a result for this year, A) $10,000 is includable in Marian's gross income. B) the $10,000 from last year's conversion is subject to the 10% penalty tax. C) $20,000 is includable in Marian's gross income. D) the $10,000 withdrawal from last year's conversion is not subject to the 10% penalty tax.

D) the $10,000 withdrawal from last year's conversion is not subject to the 10% penalty tax. First, this is a qualified distribution because the Roth IRA owner has had a Roth IRA for more than five years and she is older than 59½. Thus, there will be no tax or penalty on this distribution. Also, the conversion amounts were already included in Marian's gross income when converted. Therefore, they will not be subject to income taxes again when the converted dollars are withdrawn. The distribution allocable to the $10,000 con version contribution made last year (less than five taxable years ago) starts out as being subject to the early distribution penalty because it was withdrawn less than five years after the conversion. However, it is not subject to the 10% penalty tax under Section 72(t) in this case because Marian is over age 59½, which is one of the exceptions to the 10% penalty. The withdrawal of the conversion amount from the first conversion is not subject to the 10% early withdrawal penalty because the conversion is more than five years old. Thus, if the owner would have been younger than age 59½, the withdrawal of that money would not have been subject to the early withdrawal penalty.

Which of the following is NOT subject to 20% mandatory federal income tax withholding if a rollover is not a direct transfer? I. IRA rollovers II. Section 403(b) plan/tax-sheltered annuity (TSA) rollovers III. Money purchase pension plan rollovers IV. Profit-sharing plan rollovers

I Only Of those listed, only IRAs are not subject to 20% withholding (regardless of the type of rollover). In contrast, all qualified plan and Section 403(b) plan distributions require withholding in the absence of a direct trustee-to-trustee transfer. SEP plans, while not listed, are a type of IRA and thus do not require 20% withholding.

Joyce has decided to offer a retirement plan to her employees. She has selected a savings incentive match plan for employees (SIMPLE) and is trying to decide between a SIMPLE IRA and a SIMPLE 401(k). All of the following statements apply to both types of SIMPLEs except: I. there is a 25% penalty for early distributions from a participant's SIMPLE account within two years of entry into the plan. II. SIMPLE assets may only be rolled over into another SIMPLE within the first two years of initial participation in the plan by a participant. III. SIMPLEs are not subject to the top-heavy rules that apply to qualified plans. IV. employer contributions to a SIMPLE are not subject to payroll taxes (FICA and FUTA). A) II and III B) I only C) I, III, and IV D) II and IV

I Only Only early distributions from a SIMPLE IRA within the first two years of initial participation in the plan are subject to the 25% early withdrawal penalty.

Which of the following statements regarding a simplified employee pension (SEP) plan are CORRECT? I. Distributions used to fund college education costs for the participant's child are not subject to the 10% early withdrawal penalty. II. Distributions from a SEP plan will not be subject to the 10% early withdrawal penalty if the participant leaves the sponsoring company after attaining age 55. A) Both I and II B) I only C) Neither I nor II D) II only

I Only The exception to the early withdrawal penalty for those who leave the employer after attaining age 55 applies only to qualified plans and Section 403(b) plans. Distributions used to fund college education costs for the participant's child are not subject to the 10% early withdrawal penalty when the retirement plan is a SEP because a SEP is treated as an IRA. Also, QDROs are not allowed with a SEP because a SEP is treated as an IRA.

A Section 401(k) plan allows plan participants the opportunity to defer taxation on a portion of regular salary or bonuses simply by electing to have such amounts contributed to the plan instead of receiving them in cash. Which of these is a rule that applies to Section 401(k) plan elective deferrals? I. Section 401(k) plan elective deferrals are immediately 100% vested and cannot be forfeited. II. In-service withdrawals are to be made only if an individual has attained age 62. III. An extra nondiscrimination test called the actual deferral percentage test applies to elective deferral amounts. A) I and II B) I only C) I and III D) III only

I and III Statement II is incorrect. A Section 401(k) plan may allow in-service distributions before age 62. Age 59½ is one reason some plans allow certain in-service withdrawals.

Which of the following types of qualified retirement plans are subject to the minimum funding requirements? I. Defined benefit pension plans II. Money purchase pension plans III. Profit sharing plans IV. Target benefit plans A) I and II B) I, II, and IV C) I, III, and IV D) II, III, and IV

I, II and IV The only qualified plans exempt from the minimum funding standard are profit sharing plans and stock bonus plans. Section 401(k) provisions are only permitted with profit sharing plans and stock bonus plans.

Which of the following is CORRECT in stating the rules regarding distributions from an IRA plan? I. The annuity rules govern an IRA distribution that includes nondeductible and deductible contributions. II. At the IRA owner's death, the account balance exceeding the greater of $150,000 is included in the decedent's gross estate. III. A distribution attributable to contributions to a Roth IRA is subject to the premature distribution penalty tax, if withdrawn before age 59½. IV. Distributions related to the death of the IRA owner are exempt from the premature distribution penalty tax. A) I, II, and III B) II, III, and IV C) I, III, and IV D) I and IV

I and IV Section 72 annuity rules relating to the recovery of the participant's cost basis still apply to an IRA distribution consisting of deductible and nondeductible contributions. (The Small Business Job Protection Act of 1996 provided a simplified method of cost basis recovery for qualified plans and TSA annuities; however, the provisions for IRAs were not affected.) Death is an exception to the premature distribution penalty. Option II is wrong because the total balance of an IRA will be included in the owner's gross estate; however, if the spouse is the beneficiary, the marital deduction may be available. Option III is wrong because this type of distribution is not subject to the premature distribution penalty (10%). With a Roth IRA, nonqualifying distribution amounts that exceed contributions or include earnings are subject to both income tax and the 10% penalty.

In a Section 401(k) plan, which of these must be considered in complying with the maximum annual additions limit? I. Employee contributions II. Catch-up contributions for an employee age 50 or older III. Dividends paid on employer stock held in the Section 401(k) plan IV. Employer contributions A) I and IV B) I, II, and IV C) II and IV D) I and II

I and IV Statement I is correct. Employee contributions are counted against the annual additions limit. Statement II is incorrect. Catch-up contributions for an employee age 50 or older are not counted against the annual additions limit. Statement III is incorrect. Earnings on plan investments are not taken into account when computing the maximum annual additions limit. Statement IV is correct. For 2023, the annual additions limit is the lesser of 100% of the employee's compensation, or $66,000.

Which of the following is subject to the required minimum distribution (RMD) requirements after the account owner/plan participant dies? I. Traditional IRAs II. Roth IRAs III. Qualified plans A) I and II B) I only C) II and III D) I, II, and III

I, II and III All of these retirement accounts are subject to RMD requirements after the account owner/plan participant dies. However, RMD requirements do not apply to Roth IRAs while the owner is alive.

Which of the following plans are allowed to offer Section 401(k) provisions? I. Profit-sharing plans II. Employee stock ownership plans (ESOPs) III. Traditional defined benefit pension plans IV. Cash balance pension plans A) III and IV B) II and III C) I, II, and III D) I, II, and IV

I, II and III Defined contribution plans, such as profit-sharing plans, stock bonus plans, ESOPs, and savings plans are allowed to offer Section 401(k) provisions. A traditional defined pension benefit plan is permitted to accept Section 401(k) pretax employee contributions [DB(k) plans]. Cash balance pension plans cannot offer Section 401(k) provisions.

Which of the following are characteristics that simplified employee pensions (SEPs) share with qualified profit sharing plans? I. Limitation on employer contributions II. Application of controlled group rules III. Nondiscrimination requirements IV. Statutory eligibility requirements (age 21, one year of service) A) II, III, and IV B) I and II C) II and IV D) I, II, and III

I, II and III SEPs and profit sharing plans are both subject to the 25% limit on deductible employer contributions, the controlled group rules, and nondiscrimination requirements. Eligibility requirements are different for a SEP.

How does simplified employee pension (SEP) plan participation affect an employee's IRA contributions? I. The deductibility of an active participant's IRA contribution depends upon his MAGI. II. SEP plan participation does not reduce or eliminate an employee's ability to fund an IRA. III. Employees who participate in a SEP plan are considered active participants in an employer-sponsored retirement plan for the tax year in which an employer contribution is made. IV. Employees who participate in a SEP plan are not considered active participants in an employer retirement plan for the tax year in which an employer contribution is made. A) I, II, and III B) II and III C) I and IV D) I, II, and IV

I, II and III Statements I, II, and III are correct. Employees who participate in a SEP plan are considered active participants in an employer retirement plan for the tax year in which an employer contribution is made. The deductibility of an active participant's IRA contribution depends upon his MAGI and can be phased out or eliminated at certain income levels. SEP plan participation does not reduce or eliminate an employee's ability to fund an IRA. The IRA can be funded, but not necessarily deducted from gross income.

Which of these legal requirements apply to employee stock ownership plans (ESOPs)? I. ESOPs must permit participants who have reached age 55 and have at least 10 years of service the opportunity to diversify their accounts. II. ESOPs cannot be integrated with Social Security. III. An employer's deduction for ESOP contributions and amounts made to repay interest on an ESOP's debt cannot exceed 25% of the participants' payroll. IV. The mandatory 20% income tax withholding requirement does not apply to distributions of employer stock from an ESOP. A) I and IV B) I, II, and IV C) I, II, and III D) II and III

I, II and IV Options I and II correctly state the diversification rule and the rule that prohibits integrated ESOPs. There is no limit on amounts used to pay interest on ESOP debt. ESOP distributions of employer stock only are not subject to the 20% income tax withholding requirement.

Which of these statements regarding Social Security plan integration is false? I. Because there is a disparity in the Social Security system, all retirement plans are allowed to integrate with Social Security. II. Only the excess method can be used by a defined benefit pension plan. III. Under the offset method of integration, a fixed or formula amount reduces the plan formula. IV. The maximum increase in benefits for earnings above covered compensation level is 5.7% for a defined benefit pension plan. A) II and III B) II and IV C) I and III D) I, II, and IV

I, II and IV Statement I is incorrect because not all retirement plans are allowed to integrate with Social Security. For example, ESOPs and SARSEPs are not permitted to use integration. Statement II is incorrect because the defined benefit pension plan can use either the excess or offset method in integrating with Social Security. Only the excess method can be used with defined contribution plans. Statement IV is incorrect because the permitted disparity limit for a defined benefit pension plan is 26.25% above the covered compensation level (0.75% per year for up to 35 years).

A Section 401(k) plan allows plan participants the opportunity to defer taxation on a portion of regular salary or bonuses simply by electing to have such amounts contributed to the plan instead of receiving them in cash. Which of these statements are rules that apply to Section 401(k) plans? I. Section 401(k) elective deferrals are immediately 100% vested and cannot be forfeited. II. A Section 401(k) plan may allow in-service distributions. III. Eligible catch-up contributions are not considered in the application of the maximum annual additions limit. IV. The maximum elective deferral contribution for 2023 is $22,500, with an additional $7,500 catch-up for individuals age 50 or older. A) II, III, and IV B) I, III, and IV C) I, II, III, and IV D) I, II, and III

I, II, III and IV

Linda works for a Section 501(c)(3) organization that sponsors a Section 403(b) plan. Which of the following statements regarding a Section 403(b) plan is false? I. A special catch-up provision available for Section 403(b) plan participants allows the elective deferral amount to be doubled in the last three years before retirement. II. Funding in the plan is limited to annuity contracts or mutual funds. III. Because the organization is a nonprofit, employer contributions to the plan are currently taxable to the employees. IV. The elective deferral limit (not considering any available catch-up contribution) is $66,000 for 2023. A) III and IV B) I, II, III, and IV C) II only D) I, III, and IV

I, III, IV Statement II is correct. Funding in a Section 403(b) plan is limited to annuity contracts or mutual funds. Statements I, III, and IV are incorrect. Employer contributions to the plan are not currently taxable to the employees. The elective deferral limit is $22,500 in 2023 with an age-50-or-over catch-up contribution of $7,500. Special catch-up contribution provisions may be available to qualifying participants, but the maximum available special catch-up amount in a tax year is $3,000.

Which of these statements regarding money purchase pension plans is CORRECT? I. Generally, employer securities held by the plan cannot exceed 25% of the FMV of the plan assets at the times the securities are purchased. II. The plan participant can easily understand the plan's simple design, and contributions are based on the participant's salary for each year of his career, rather than on salary at retirement.

II Only

Which of these are minimum coverage tests for all qualified retirement plans? I. Minimum benefit test II. Average benefits percentage test III. Ratio test IV. 50/40 test A) I and II B) I and III C) II and III D) II, III, and IV

II and III The two minimum coverage tests for qualified retirement plans are the average benefits percentage test and the ratio test. To be qualified, a retirement plan must meet at least one of these tests if the plan does not meet the percentage (safe harbor) test. The 50/40 test is only required for defined benefit plans. There is no such thing as a minimum benefit test.

Jane, age 56, has $500,000 in a traditional IRA rollover account from her previous employer's profit-sharing plan. She also receives a monthly retirement pension from her previous employer's qualified plan of $30,000 per year. Jane needs an extra $20,000 per year from her IRA to meet her living expenses until her Social Security payments begin at age 62. Which of the following is CORRECT? I. Since Jane is under age 59½, she cannot avoid having to pay a mandatory 10% penalty tax on any amounts withdrawn from her IRA. II. Jane will have to report the $20,000 withdrawal as ordinary income on her personal income tax return. III. Jane can avoid the 10% penalty tax if she bases her withdrawal on the "substantially equal payment" method available to individuals under age 59½. IV. The $20,000 per year withdrawal from Jane's IRA will be subject to the 15% excise tax on "excess distributions" from qualified plans. A) II on

II and III While taxpayers under age 59½ may pay a penalty tax on IRA distributions, there are methods of avoiding the penalty, as is accurately stated in Statement III. The income from the IRA would be fully taxable as ordinary income on her return. There is no 15% excise tax on "excess distributions."

Fred has been researching IRAs and learning of the advantages and disadvantages of using an IRA as a retirement savings vehicle. Which of these regarding an IRA is CORRECT? I. When the investments in an IRA consist solely of securities, net unrealized appreciation (NUA) treatment of a lump-sum distribution is available. II. In 2023, eligible individuals may contribute up to $6,500 to an IRA and an additional $1,000 when age 50 or over. III. Certain taxpayers may be eligible for an income tax credit for contributions to a traditional IRA but not a Roth IRA. IV. Earnings on assets held in an IRA are not subject to federal income tax until withdrawn from the account.

II and IV

Which of these are CORRECT statements about defined contribution pension plans? I. They use an indefinite allocation formula. II. They provide a benefit that is based on the value of a participant's account. III. They require employer contributions to be made from business earnings. IV. They require fixed employer contributions according to the terms of the plan. A) II and IV B) I and II C) II and III D) I and IV

II and IV Employer contributions to a defined contribution pension plan (money purchase plan or target benefit plan) must be fixed according to the terms of the plan; e.g., a plan could stipulate that the employer shall contribute an amount equal to 10% of the compensation of each participant. The investment performance of a participant's account balance determines the value of his or her benefit upon termination or retirement. However, contributions must be made to fund a pension plan, even if an employer has no earnings or profits. Defined contribution plans must also have definitely determinable benefits; i.e., the annual employer contribution to an employee's account must be fixed or definitely determinable.

Under which of the following circumstances is a target benefit plan the most appropriate choice for small-business owners? I. To simplify and reduce the cost of eliminating a defined benefit plan (without termination) by amending it into a target plan II. Where the employer wants to provide larger retirement benefits for key employees who are significantly older than the other employees III. To meet the employer's goal of maximizing deductible contributions to provide benefits for older, highly compensated employees IV. Where the employer is opposed to assuming the investment risk and prefers the simplicity of a separate account plan A) II, III, and IV B) I, II, and IV C) II and IV D) I and III

II and IV Statements II and IV are correct for the following reasons. Target benefit plans have benefit formulas similar to those of defined benefit plans, which favor employees who are significantly older and higher paid than the average employee of the employee group. Because target benefit plans use separate accounts, the participants bear the risk of the plan's investment performance. Statement I is incorrect because amending a defined benefit plan into a target plan will result in termination of the defined benefit plan. Statement III is not true since a DC plan is limited to 25%, and a DB plan is not limited by a set percentage. Therefore, the DB plan will usually provide a greater tax deduction if an age-weighted plan works best. The key word to Statement III is "maximizing."

Which of the following are CORRECT statements pertaining to IRA rollover requirements? I. If an IRA account is distributed directly to the IRA participant, the original IRA custodian must withhold 20% from the proceeds. II. An IRA rollover must be completed within 60 days following the distribution date. III. An IRA account may be rolled over to another IRA once a year. IV. Direct trustee-to-trustee IRA transfers may be made as often as the IRA owner wishes.

II, III and IV Once a year, an IRA owner may roll over his account to a new IRA within 60 days after distribution from the original IRA. Direct trustee-to-trustee transfers may be made as often as desired. The 20% withholding requirement does not apply to IRA distributions; the 20% mandatory withholding applies to eligible rollover distributions made directly to the plan participant from employer-sponsored qualified retirement plans or TSAs.

Which of the following are characteristics of a Section 403(b) plan (TSA)? I. An employee, age 52, has a maximum contribution of $22,500 for 2023. II. Qualifying lump-sum distributions from a TSA are eligible for NUA tax treatment. III. Rollovers to IRAs are permitted. A) I and II B) I and III C) II and III D) III only

III Only For 2023, the regular elective deferral limit for employee elective deferral contributions to a Section 403(b) plan is $22,500. Employees over age 50 would be allowed to defer an additional $7,500 (2023) as a catch-up contribution. An employee who has worked for an eligible plan sponsor for at least 15 years and has not maximized deferrals to the Section 403(b) plan in past years is also eligible for an additional $3,000 catch-up contribution. The eligible plan sponsors are health, educational, and church employers. With this, rollovers are allowable, and NUA tax treatment is not permitted for Section 403(b) plans because a not-for-profit employer does not have employer stock to distribute.

Which of the following is true regarding the interest rate credit used in cash balance pension plans? I. The interest rate credited to a participant's hypothetical account is determined upon the establishment of the plan and cannot fluctuate. II. If the underlying investments of the plan outperform the interest rate credit guarantee in a given year, the participant will receive a greater credit for that given year. III. If the underlying investments of the plan outperform the interest rate credit guarantee in a given year, the employer may reduce plan contributions for that given year. IV. Because of the hypothetical individual accounts, plan participants may choose among various fixed interest rate investments for their accounts. A) I, II, and IV B) III only C) I and III D) IV only

III Only Only Statement III is correct. Statement I is incorrect because the interest rate credit may be linked to a market rate, such as a Treasury security; the rate credited may fluctuate in a given year, but will never be less than the stated formula. Statement II is incorrect because the employee does not receive additional interest credit if the underlying assets outperform the guaranteed credit. This also makes it possible for the employer to reduce plan contributions in years in which the underlying assets outperform the guaranteed interest credit. The employer bears the investment risk in a cash balance pension plan; participants do not select their own investments.

Joe, 46, has owned his company for 18 years and wishes to retire at age 70. All of Joe's employees are older than he is and have an average length of service with the company of eight years. Joe would like to adopt a qualified retirement plan that would favor him and reward employees who have rendered long service. Joe has selected a traditional defined benefit pension plan with a unit benefit formula. Which of these statements regarding Joe's traditional defined benefit pension plan is CORRECT? I. Increased profitability would increase both Joe's and his employees' pension contributions. II. A unit benefit plan formula allows for higher levels of integration than other defined benefit pension plans. III. A unit benefit plan formula rewards older employees who were hired in their 50s or 60s. IV. A traditional defined benefit pension plan will maximize Joe's benefits and reward long-term employees based on length of

IV Only Statement I is incorrect. Contributions to traditional defined benefit pension plans are not dependent on the profitability of a company. Statement II is incorrect because a unit benefit plan formula will not allow higher integration levels. Statement III is incorrect because a flat percentage formula favors workers without much longevity.


संबंधित स्टडी सेट्स

TEST #1- Ch. 12,13,14 (Business Communication/ UCA Carson Fall 2019)

View Set

APUSH Unit 5 College Board Review Questions

View Set

RPRACTICES16: Taxes, Tax Year, Prop 13, Capitol Gains, Determining a Profit or Loss, Depreciation

View Set

Chapter 13- Nervous System: The Brain & Cranial Nerves

View Set

Chapter 9 Real Estate and Other Assets

View Set

Altered Intracranial Regulation - Nursing Care: Altered Intracranial Regulation

View Set

PEDS Chapter 48: Nursing Care of the Child With an Alteration in Metabolism/Endocrine Disorder

View Set